You are on page 1of 124

Sixth Term Examination Papers

MATHEMATICS 1
MONDAY 15 JUNE 2015

9465
Morning
Time: 3 hours

  

Additional Materials: Answer Booklet


Formulae Booklet

INSTRUCTIONS TO CANDIDATES
Please read this page carefully, but do not open this question paper until you are
told that you may do so.
Write your name, centre number and candidate number in the spaces on the answer
booklet.
Begin each answer on a new page.
Write the numbers of the questions you answer in the order attempted on the front of the
answer booklet.
INFORMATION FOR CANDIDATES
Each question is marked out of 20. There is no restriction of choice.
All questions attempted will be marked.
Your final mark will be based on the six questions for which you gain the highest marks.
You are advised to concentrate on no more than six questions. Little credit will be given
for fragmentary answers.
You are provided with a Mathematical Formulae Booklet.
Calculators are not permitted.

Please wait to be told you may begin before turning this page.
_____________________________________________________________________________

This question paper consists of 7 printed pages and 1 blank page.


UCLES 2015

Section A:

(i)

Pure Mathematics

Sketch the curve y = ex (2x2 5x + 2) .


Hence determine how many real values of x satisfy the equation ex (2x2 5x + 2) = k
in the dierent cases that arise according to the value of k.
You may assume that xn ex 0 as x for any integer n.

(ii)

Sketch the curve y = ex (2x4 5x2 + 2) .

cos 15

3+1

and nd a similar expression for sin 15 .


2 2

(i)

Show that

(ii)

Show that cos is a root of the equation

4x3 3x cos 3 = 0 ,
and nd the other two roots in terms of cos and sin .
(iii) Use parts (i) and (ii) to solve the equation y 3 3y
surd form.

2 = 0 , giving your answers in

A prison consists of a square courtyard of side b bounded by a perimeter wall and a square
building of side a placed centrally within the courtyard. The sides of the building are parallel
to the perimeter walls.
Guards can stand either at the middle of a perimeter wall or in a corner of the courtyard. If
the guards wish to see as great a length of the perimeter wall as possible, determine which
of these positions is preferable. You should consider separately the cases b < 3a and b > 3a .

2
UCLES 2015

The midpoint of a rod of length 2b slides on the curve y = 14 x2 , x  0, in such a way that
the rod is always tangent, at its midpoint, to the curve. Show that the curve traced out by
one end of the rod can be written in the form
x = 2 tan b cos
y = tan2 b sin
for some suitably chosen angle which satises 0  < 12 .
When one end of the rod is at a point A on the y-axis, the midpoint is at point P and = .
Let R be the region bounded by the following:
the curve y = 14 x2 between the origin and P ;
the y-axis between A and the origin;
the half-rod AP .
Show that the area of R is

(i)

2
3

tan3 .

The function f is dened, for x > 0, by




f(x) =
1

(t 1)x1 dt .

By evaluating the integral, sketch the curve y = f(x).


(ii)

The function g is dened, for < x < , by




g(x) =
1

1
dt .
1 2xt + x2

By evaluating the integral, sketch the curve y = g(x).

The vertices of a plane quadrilateral are labelled A, B, A and B  , in clockwise order. A


point O lies in the same plane and within the quadrilateral. The angles AOB and A OB 
are right angles, and OA = OB and OA = OB  .
Use position vectors relative to O to show that the midpoints of AB, BA , A B  and B  A
are the vertices of a square.
Given that the lengths of OA and OA are xed (and the conditions of the rst paragraph
still hold), nd the value of angle BOA for which the area of the square is greatest.

3
UCLES 2015

[Turn over

Let

f(x) = 3ax2 6x3

and, for each real number a, let M(a) be the greatest value of f(x) in the interval 13  x  1.
Determine M(a) for a  0. [The formula for M(a) is dierent in dierent ranges of a; you
will need to identify three ranges.]

Show that:
(i)

1 + 2 + 3 + + n = 12 n(n + 1);

(ii)

if N is a positive integer, m is a non-negative integer and k is a positive odd integer,


then (N m)k + mk is divisible by N .

Let S = 1k + 2k + 3k + + nk , where k is a positive odd integer. Show that if n is odd then


S is divisible by n and that if n is even then S is divisible by 12 n.
Show further that S is divisible by 1 + 2 + 3 + + n.

4
UCLES 2015

Section B:

Mechanics

A short-barrelled machine gun stands on horizontal ground. The gun res bullets, from

ground level, at speed u continuously from t = 0 to t =


, where is a positive constant,
6
but does not re outside this time period. During this time period, the angle of elevation
of the barrel decreases from 13 to 16 and is given at time t by
= 13 t .
g
. Show that, in the case
2u
at a distance
Let k =

1
2

k

1
2

3, the last bullet to hit the ground does so

2ku2 1 k 2
g

from the gun.


What is the corresponding result if k < 12 ?

10

A bus has the shape of a cuboid of length a and height h. It is travelling northwards on a
journey of xed distance at constant speed u (chosen by the driver). The maximum speed of
the bus is w. Rain is falling from the southerly direction at speed v in straight lines inclined
to the horizontal at angle , where 0 < < 12 .
By considering rst the case u = 0, show that for u > 0 the total amount of rain that hits
the roof and the back or front of the bus in unit time is proportional to


hv cos u + av sin .
Show that, in order to encounter as little rain as possible on the journey, the driver should
choose u = w if either w < v cos or a sin > h cos . How should the speed be chosen if
w > v cos and a sin < h cos ? Comment on the case a sin = h cos .
How should the driver choose u on the return journey?

5
UCLES 2015

[Turn over

11

Two long circular cylinders of equal radius lie in equilibrium on an inclined plane, in contact
with one another and with their axes horizontal. The weights of the upper and lower
cylinders are W1 and W2 , respectively, where W1 > W2 . The coecients of friction between
the inclined plane and the upper and lower cylinders are 1 and 2 , respectively, and the
coecient of friction between the two cylinders is . The angle of inclination of the plane
is (which is positive).
(i)

Let F be the magnitude of the frictional force between the two cylinders, and let F1
and F2 be the magnitudes of the frictional forces between the upper cylinder and the
plane, and the lower cylinder and the plane, respectively. Show that F = F1 = F2 .

(ii)

Show that

and that
tan 

21 W1
.
(1 + 1 )(W1 + W2 )

6
UCLES 2015

W1 + W2
,
W1 W2

Section C:

12

Probability and Statistics

The number X of casualties arriving at a hospital each day follows a Poisson distribution
with mean 8; that is,
P(X = n) =

e8 8n
,
n!

n = 0, 1, 2, . . . .

Casualties require surgery with probability 14 . The number of casualties arriving on any
given day is independent of the number arriving on any other day and the casualties require
surgery independently of one another.
(i)

What is the probability that, on a day when exactly n casualties arrive, exactly r of
them require surgery?

(ii)

Prove (algebraically) that the number requiring surgery each day also follows a Poisson
distribution, and state its mean.

(iii) Given that in a particular randomly chosen week a total of 12 casualties require surgery
on Monday and Tuesday, what is the probability that 8 casualties require surgery on
Monday? You should give your answer as a fraction in its lowest terms.

13

A fair die with faces numbered 1, . . . , 6 is thrown repeatedly. The events A, B, C, D and E
are dened as follows.
A: the rst 6 arises on the nth throw.
B: at least one 5 arises before the rst 6.
C: at least one 4 arises before the rst 6.
D: exactly one 5 arises before the rst 6.
E: exactly one 4 arises before the rst 6.
Evaluate the following probabilities:
(i) P(A)

(ii) P(B)

(iii) P(B C)

(iv) P(D)

(v) P(D E) .

For some parts of this question, you may want to make use of the binomial expansion in the
form:
n(n + 1) 2
(n + r 1)! r
(1 x)n = 1 + nx +
x + +
x + .
2
r!(n 1)!

7
UCLES 2015

[Turn over

BLANK PAGE

UCLES 2015

Sixth Term Examination Papers


MATHEMATICS 2
WEDNESDAY 17 JUNE 2015

9470
Morning
Time: 3 hours

 

Additional Materials: Answer Booklet


Formulae Booklet

INSTRUCTIONS TO CANDIDATES
Please read this page carefully, but do not open this question paper until you are
told that you may do so.
Write your name, centre number and candidate number in the spaces on the answer
booklet.
Begin each answer on a new page.
Write the numbers of the questions you answer in the order attempted on the front of the
answer booklet.
INFORMATION FOR CANDIDATES
Each question is marked out of 20. There is no restriction of choice.
All questions attempted will be marked.
Your final mark will be based on the six questions for which you gain the highest marks.
You are advised to concentrate on no more than six questions. Little credit will be given
for fragmentary answers.
You are provided with a Mathematical Formulae Booklet.
Calculators are not permitted.

Please wait to be told you may begin before turning this page.
_____________________________________________________________________________

This question paper consists of 8 printed pages and 4 blank pages.


UCLES 2015

BLANK PAGE

Section A:

(i)

Pure Mathematics

By use of calculus, show that x ln(1 + x) is positive for all positive x. Use this result
to show that
n

1
> ln(n + 1) .
k
k=1

(ii)

By considering x + ln(1 x), show that


1
< 1 + ln 2 .
k2
k=1

In the triangle ABC, angle BAC = and angle CBA = 2, where 2 is acute, and BC = x.
Show that AB = (3 4 sin2 )x.
The point D is the midpoint of AB and the point E is the foot of the perpendicular from C
to AB. Find an expression for DE in terms of x.
The point F lies on the perpendicular bisector of AB and is a distance x from C. The points
F and B lie on the same side of the line through A and C. Show that the line F C trisects
the angle ACB.

Three rods have lengths a, b and c, where a < b < c. The three rods can be made into a
triangle (possibly of zero area) if a + b  c.
Let Tn be the number of triangles that can be made with three rods chosen from n rods of
lengths 1, 2, 3, . . . , n (where n  3). Show that T8 T7 = 2 + 4 + 6 and evaluate T8 T6 .
Write down expressions for T2m T2m1 and T2m T2m2 .
Prove by induction that T2m = 61 m(m 1)(4m + 1) , and nd the corresponding result for
an odd number of rods.

2
UCLES 2015

(i)

The continuous function f is dened by


( < x < )

tan f(x) = x
and f(0) = . Sketch the curve y = f(x) .
(ii)

The continuous function g is dened by


x
1 + x2

tan g(x) =

and g(0) = . Sketch the curves y =

( < x < )

x
and y = g(x) .
1 + x2

(iii) The continuous function h is dened by h(0) = and


tan h(x) =

x
1 x2

(x = 1) .

(The values of h(x) at x = 1 are such that h(x) is continuous at these points.) Sketch
the curves y =

x
and y = h(x).
1 x2

In this question, the arctan function satises 0  arctan x < 12 for x  0 .


(i)

Let
Sn =

n



arctan

m=1

1
2m2


,

for n = 1, 2, 3, . . . . Prove by induction that


tan Sn =

n
.
n+1

Prove also that


Sn = arctan
(ii)

n
.
n+1

In a triangle ABC, the lengths of the sides AB and BC are 4n2 and 4n4 1, respectively,
and the angle at B is a right angle. Let angle BCA = 2n . Show that


n=1

n = 14 .

3
UCLES 2015

[Turn over

(i)

Show that
sec2
Hence integrate

(ii)


1

x
=
4
2

1

2
.
1 + sin x

1
with respect to x.
1 + sin x

By means of the substitution y = x, show that





xf(sin x) dx =
f(sin x) dx,
2 0
0
where f is any function for which these integrals exist.
Hence evaluate

(iii) Evaluate

x
dx .
1 + sin x

2x3 3x2
dx.
(1 + sin x)2

A circle C is said to be bisected by a curve X if X meets C in exactly two points and these
points are diametrically opposite each other on C.
(i)

Let C be the circle of radius a in the x-y plane with centre at the origin. Show, by
giving its equation, that it is possible to nd a circle of given radius r that bisects C
provided r > a. Show that no circle of radius r bisects C if r  a .

(ii)

Let C1 and C2 be circles with centres at (d, 0) and (d, 0) and radii a1 and a2 , respectively, where d > a1 and d > a2 . Let D be a circle of radius r that bisects both C1
a2 a21
.
and C2 . Show that the x-coordinate of the centre of D is 2
4d
Obtain an expression in terms of d, r, a1 and a2 for the y-coordinate of the centre
of D, and deduce that r must satisfy



16r2 d2  4d2 + (a2 a1 )2 4d2 + (a2 + a1 )2 .

4
UCLES 2015

8
L

C1
C2
L

The diagram above shows two non-overlapping circles C1 and C2 of dierent sizes. The lines
L and L are the two common tangents to C1 and C2 such that the two circles lie on the
same side of each of the tangents. The lines L and L intersect at the point P which is called
the focus of C1 and C2 .
(i)

Let x1 and x2 be the position vectors of the centres of C1 and C2 , respectively. Show
that the position vector of P is
r 1 x 2 r2 x 1
,
r1 r2
where r1 and r2 are the radii of C1 and C2 , respectively.

(ii)

The circle C3 does not overlap either C1 or C2 and its radius, r3 , satises r1 = r3 = r2 .
The focus of C1 and C3 is Q, and the focus of C2 and C3 is R. Show that P , Q and R
lie on the same straight line.

(iii) Find a condition on r1 , r2 and r3 for Q to lie half-way between P and R.

5
UCLES 2015

[Turn over

Section B:

10

Mechanics

An equilateral triangle ABC is made of three light rods each of length a. It is free to rotate in
a vertical plane about a horizontal axis through A. Particles of mass 3m and 5m are attached
to B and C respectively. Initially, the system hangs in equilibrium with BC below A.
(i)

Show that, initially, the angle that BC makes with the horizontal is given by sin = 17 .

(ii)

The triangle receives an impulse that imparts a speed v to the particle B. Find the
minimum speed v0 such that the system will perform complete rotations if v > v0 .

A particle of mass m is pulled along the oor of a room in a straight line by a light string
which is pulled at constant speed V through a hole in the ceiling. The oor is smooth and
horizontal, and the height of the room is h. Find, in terms of V and , the speed of the
particle when the string makes an angle of with the vertical (and the particle is still in
contact with the oor). Find also the acceleration, in terms of V , h and .
Find the tension in the string and hence show that the particle will leave the oor when
tan4 =

6
UCLES 2015

V2
.
gh

11

Three particles, A, B and C, each of mass m, lie on a smooth horizontal table. Particles A
and C are attached to the two ends of a light inextensible string of length 2a and particle B
is attached to the midpoint of the string. Initially, A, B and C are at rest at points (0, a),
(0, 0) and (0, a), respectively.
An impulse is delivered to B, imparting to it a speed u in the positive x direction. The string
remains taut throughout the subsequent motion.

y
A

B
x

(i)

At time t, the angle between the x-axis and the string joining A and B is , as shown
in the diagram, and B is at (x, 0). Write down the coordinates of A in terms of x, a
and at this time. Given that the velocity of B is (v, 0), show that the velocity of A
is (v + a sin , a cos ), where the dot denotes dierentiation with respect to time.

(ii)

Show that, before A and C rst collide,


3v + 2a sin = u

and

2 =

u2
.
a2 (3 2 sin2 )

(iii) When A and C collide, the collision is elastic (no energy is lost). At what value of
does the second collision between particles A and C occur? (You should justify your
answer.)
(iv) When v = 0, what are the possible values of ? Is v = 0 whenever takes these values?

7
UCLES 2015

[Turn over

Section C:

12

Probability and Statistics

Four players A, B, C and D play a coin-tossing game with a fair coin. Each player chooses
a sequence of heads and tails, as follows:
Player A: HHT;

Player B: THH;

Player C: TTH;

Player D: HTT.

The coin is then tossed until one of these sequences occurs, in which case the corresponding
player is the winner.
1
4

(i)

Show that, if only A and B play, then A has a probability of

(ii)

If all four players play together, nd the probabilities of each one winning.

of winning.

(iii) Only B and C play. What is the probability of C winning if the rst two tosses are TT?
Let the probabilities of C winning if the rst two tosses are HT, TH and HH be p, q
and r, respectively. Show that p = 12 + 12 q.
Find the probability that C wins.

13

The maximum height X of ood water each year on a certain river is a random variable with
probability density function f given by

ex for x  0 ,
f(x) =
0
otherwise,
where is a positive constant.
It costs ky pounds each year to prepare for ood water of height y or less, where k is a
positive constant and y  0. If X  y no further costs are incurred but if X > y the
additional cost of ood damage is a(X y) pounds where a is a positive constant.
(i)

Let C be the total cost of dealing with the oods in the year. Show that the expectation
of C is given by
a
E(C) = ky + ey .

How should y be chosen in order to minimise E(C), in the dierent cases that arise
according to the value of a/k?

(ii)

Find the variance of C, and show that the more that is spent on preparing for ood
water in advance the smaller this variance.

8
UCLES 2015

BLANK PAGE

UCLES 2015

BLANK PAGE

UCLES 2015

BLANK PAGE

UCLES 2015

Sixth Term Examination Papers


MATHEMATICS 3
FRIDAY 19 JUNE 2015

9475
Morning
Time: 3 hours

   

Additional Materials: Answer Booklet


Formulae Booklet

INSTRUCTIONS TO CANDIDATES
Please read this page carefully, but do not open this question paper until you are
told that you may do so.
Write your name, centre number and candidate number in the spaces on the answer
booklet.
Begin each answer on a new page.
Write the numbers of the questions you answer in the order attempted on the front of the
answer booklet.
INFORMATION FOR CANDIDATES
Each question is marked out of 20. There is no restriction of choice.
All questions attempted will be marked.
Your final mark will be based on the six questions for which you gain the highest marks.
You are advised to concentrate on no more than six questions. Little credit will be given
for fragmentary answers.
You are provided with a Mathematical Formulae Booklet.
Calculators are not permitted.

Please wait to be told you may begin before turning this page.
_____________________________________________________________________________

This question paper consists of 9 printed pages and 3 blank pages.


UCLES 2015

Section A:

(i)

Pure Mathematics

Let


In =

1
du ,
(1 + u2 )n

where n is a positive integer. Show that


In In+1 =
and deduce that
In+1 =
(ii)

Let

J=
0

1
In
2n

(2n)!
22n+1 (n!)2



f (x x1 )2 dx ,

where f is any function for which the integral exists. Show that





 


1
x2 f (x x1 )2 dx =
(1 + x2 )f (x x1 )2 dx =
f u2 du .
J=
2 0
0
0
(iii) Hence evaluate

x2n2
dx ,
(x4 x2 + 1)n

where n is a positive integer.

2
UCLES 2015

If s1 , s2 , s3 , . . . and t1 , t2 , t3 , . . . are sequences of positive numbers, we write


(sn )  (tn )
to mean
there exists a positive integer m such that sn  tn whenever n  m.
Determine whether each of the following statements is true or false. In the case of a true
statement, you should give a proof which includes an explicit determination of an appropriate
m; in the case of a false statement, you should give a counterexample.
(i)

(1000n)  (n2 ) .

(ii)

If it is not the case that (sn )  (tn ), then it is the case that (tn )  (sn ) .

(iii) If (sn )  (tn ) and (tn )  (un ), then (sn )  (un ) .


(iv) (n2 )  (2n ) .

In this question, r and are polar coordinates with r  0 and <  , and a and b are
positive constants.
Let L be a xed line and let A be a xed point not lying on L. Then the locus of points that
are a xed distance (call it d) from L measured along lines through A is called a conchoid of
Nicomedes.
(i)

Show that if
|r a sec | = b ,

()

where a > b, then sec > 0. Show that all points with coordinates satisfying () lie on
a certain conchoid of Nicomedes (you should identify L, d and A). Sketch the locus of
these points.
(ii)

In the case a < b, sketch the curve (including the loop for which sec < 0) given by
|r a sec | = b .
Find the area of the loop in the case a = 1 and b = 2.

[Note: sec d = ln | sec + tan | + C .]

3
UCLES 2015

[Turn over

(i)

If a, b and c are all real, show that the equation


z 3 + az 2 + bz + c = 0

()

has at least one real root.


(ii)

Let
S1 = z 1 + z 2 + z 3 ,

S2 = z12 + z22 + z32 ,

S3 = z13 + z23 + z33 ,

where z1 , z2 and z3 are the roots of the equation (). Express a and b in terms of S1
and S2 , and show that
6c = S13 + 3S1 S2 2S3 .
(iii) The six real numbers rk and k (k = 1, 2, 3), where rk > 0 and < k < , satisfy
3

k=1

rk sin(k ) = 0 ,

3

k=1

rk2 sin(2k ) = 0 ,

3

k=1

rk3 sin(3k ) = 0 .

Show that k = 0 for at least one value of k.


Show further that if 1 = 0 then 2 = 3 .

(i)

In the following argument to show that


steps 3, 5 and 6.

1. Assume that 2 is rational.

2 is irrational, give proofs appropriate for

2. Dene the set S to be the set of positive integers with the following property:

n is in S if and only if n 2 is an integer.


3. Show that the set S contains at least one positive integer.
4. Dene the integer k to be the smallest positive integer in S.

5. Show that ( 2 1)k is in S.

6. Show that steps 4 and 5 are contradictory and hence that 2 is irrational.
(ii)

Prove that 2 3 is rational if and only if 2 3 is rational.


1

Use an argument similar to that of part (i) to prove that 2 3 and 2 3 are irrational.

4
UCLES 2015

(i)

Let w and z be complex numbers, and let u = w + z and v = w2 + z 2 . Prove that w


and z are real if and only if u and v are real and u2  2v.

(ii)

The complex numbers u, w and z satisfy the equations


w+zu=0
2

w + z 2 u2 = 32

w3 + z 3 u =
where is a positive real number. Show that for all values of except one (which you
should nd) there are three possible values of u, all real.
Are w and z necessarily real? Give a proof or counterexample.

An operator D is dened, for any function f, by


Df(x) = x

df(x)
.
dx

The notation Dn means that D is applied n times; for example




df(x)
d
2
x
.
D f(x) = x
dx
dx
Show that, for any constant a, D2 xa = a2 xa .
(i)

Show that if P(x) is a polynomial of degree r (where r  1) then, for any positive
integer n, Dn P(x) is also a polynomial of degree r.

(ii)

Show that if n and m are positive integers with n < m, then Dn (1 x)m is divisible
by (1 x)mn .

(iii) Deduce that, if m and n are positive integers with n < m, then
m

r=0

 
m n
r = 0.
(1)
r
r

5
UCLES 2015

[Turn over

(i)

Show that under the changes of variable x = r cos and y = r sin , where r is a
function of with r > 0, the dierential equation
(y + x)
becomes

dy
=yx
dx

dr
+ r = 0.
d

Sketch a solution in the x-y plane.


(ii)

Show that the solutions of


 dy

= y x y(x2 + y 2 )
y + x x(x2 + y 2 )
dx
can be written in the form
r2 =

1
1 + Ae2

and sketch the dierent forms of solution that arise according to the value of A.

6
UCLES 2015

Section B:

Mechanics

A particle P of mass m moves on a smooth xed straight horizontal rail and is attached to
a xed peg Q by a light elastic string of natural length a and modulus . The peg Q is a
distance a from the rail. Initially P is at rest with P Q = a.
An impulse imparts to P a speed v along the rail. Let x be the displacement at time t of P
from its initial position. Obtain the equation
x 2 = v 2 k 2

x2 + a2 a

where k 2 = /(ma), k > 0 and the dot denotes dierentiation with respect to t.
Find, in terms of k, a and v, the greatest value, x0 , attained by x. Find also the acceleration
of P at x = x0 .
Obtain, in the form of an integral, an expression for the period of the motion. Show that, in
the case v  ka (that is, v is much less than ka), this is approximately


32a 1
1

du .
kv 0
1 u4

10

A light rod of length 2a has a particle of mass m attached to each end and it moves in a
vertical plane. The midpoint of the rod has coordinates (x, y), where the x-axis is horizontal
(within the plane of motion) and y is the height above a horizontal table. Initially, the rod
is vertical, and at time t later it is inclined at an angle to the vertical.
Show that the velocity of one particle can be written in the form


x + a cos
y a sin
and that




 

sin
0
x
+ a cos a2 sin
= T
mg
m
cos
1
y a sin a2 cos

where the dots denote dierentiation with respect to time t and T is the tension in the rod.
Obtain the corresponding equations for the other particle.
Deduce that x
= 0, y = g and = 0.
Initially, the midpoint of the rod is a height h above the table, the velocity of the higher

0
particle is
, and the velocity of the lower particle is
. Given that the two particles
v
v
hit the table for the rst time simultaneously, when the rod has rotated by 12 , show that
2hu2 = 2 a2 g 2uva .

7
UCLES 2015

[Turn over

11

(i)

A horizontal disc of radius r rotates about a vertical axis through its centre with
angular speed . One end of a light rod is xed by a smooth hinge to the edge of
the disc so that it can rotate freely in a vertical plane through the centre of the disc.
A particle P of mass m is attached to the rod at a distance d from the hinge. The
rod makes a constant angle with the upward vertical, as shown in the diagram, and
d sin < r.

By considering moments about the hinge for the (light) rod, show that the force exerted
on the rod by P is parallel to the rod.
Show also that
r cot = a + d cos ,
g
where a = 2 . State clearly the direction of the force exerted by the hinge on the rod,

and nd an expression for its magnitude in terms of m, g and .


(ii)

The disc and rod rotate as in part (i), but two particles (instead of P ) are attached to
the rod. The masses of the particles are m1 and m2 and they are attached to the rod
at distances d1 and d2 from the hinge, respectively. The rod makes a constant angle
with the upward vertical and d1 sin < d2 sin < r. Show that satises an equation
of the form
r cot = a + b cos ,
where b should be expressed in terms of d1 , d2 , m1 and m2 .

8
UCLES 2015

Section C:

12

Probability and Statistics

A 6-sided fair die has the numbers 1, 2, 3, 4, 5, 6 on its faces. The die is thrown n times,
the outcome (the number on the top face) of each throw being independent of the outcome
of any other throw. The random variable Sn is the sum of the outcomes.
(i)

The random variable Rn is the remainder when Sn is divided by 6. Write down the
probability generating function, G(x), of R1 and show that the probability generating
function of R2 is also G(x). Use a generating function to nd the probability that Sn
is divisible by 6.

(ii)

The random variable Tn is the remainder when Sn is divided by 5. Write down the
probability generating function, G1 (x), of T1 and show that G2 (x), the probability
generating function of T2 , is given by
G2 (x) =

2
1
36 (x

+ 7y)

where y = 1 + x + x2 + x3 + x4 .
Obtain the probability generating function of Tn and hence show that the probability
that Sn is divisible by 5 is


1
1
1 n
5
6
if n is not divisible by 5. What is the corresponding probability if n is divisible by 5?

13

Each of the two independent random variables X and Y is uniformly distributed on the
interval [0, 1].
(i)

By considering the lines x + y = constant in the x-y plane, nd the cumulative distribution function of X + Y .
Hence show that the probability density function f of (X + Y )1 is given by

2
3 for 1  t  1

2t t
2
f(t) = t3
for 1  t <

0
otherwise.

Evaluate E

(ii)

1
.
X +Y

Find the cumulative distribution function of Y /X and use this result to nd the probX
.
ability density function of
X +Y

X
Write down E
and verify your result by integration.
X +Y
9

UCLES 2015

BLANK PAGE

UCLES 2015

BLANK PAGE

UCLES 2015

BLANK PAGE

UCLES 2015

STEP Mark Scheme 2015


Mathematics
STEP 9465/9470/9475
October 2015

This mark scheme is published as an aid to teachers and students, to indicate the
requirements of the examination. It shows the basis on which marks were awarded
by the Examiners and shows the main valid approaches to each question. It is
recognised that there may be other approaches and if a different approach was
taken in the exam these were marked accordingly after discussion by the marking
team. These adaptations are not recorded here.
All Examiners are instructed that alternative correct answers and unexpected
approaches in candidates scripts must be given marks that fairly reflect the relevant
knowledge and skills demonstrated.
Mark schemes should be read in conjunction with the published question papers and
the Report on the Examination.
The Admissions Testing Service will not enter into any discussion or correspondence
in connection with this mark scheme.

UCLES 2015
More information about STEP can be found at:
www.stepmathematics.org.uk

Contents
STEP Mathematics (9465, 9470, 9475)
Mark Scheme
STEP Mathematics I
STEP Mathematics II
STEP Mathematics III

Page
4
18
45

SI-2015/Q1
(i)
y = ex(2x 1)(x 2)
( 12 , 0) & (2, 0)
dy
= ex(2x2 x 3)
dx
= ex(2x 3)(x + 1)

( 32 , e1.5) & (1, 9e 1)

B1
B1

Correct factorisation of quadratic term (or formula, etc.)


Noted or shown on sketch

M1

Derivative attempted and equated to zero for TPs

A1 A1 Noted or shown on sketch


(if y-coords. missing, allow one A1 for 2 correct x-coords.)

G1

Generally correct shape

G1

for (0, 2) noted or shown on sketch

G1

for negative-x-axis asymptote


(penalise curves that clearly turn up away from axis
or that do not actually seem to approach it)

Give M1 for either 0, 1, 2 or 3 solutions OR clear indication they know these arise from where a
horizontal line meets the curve (e.g. by a line on their diagram) implied by any correct answer(s)
Then y = k has

NO solutions for k < e1.5

A1

ONE solution for k = e1.5 and k > 9e 1

A1 A1

TWO solutions for e1.5 < k 0 and k = 9e 1

A1 A1

THREE solutions for 0< k < 9e 1

A1

FT from their y-coords.of the Max. &Min. points.

(ii)

G1

Any curve clearly symmetric in y-axis

G1

Shape correct

G1

A Max. TP at (0, 2) FT

G1

Min. TPs at (

3
2

, e1.5) FT

G1

Zeroes at x =

1
2

, 2 FT

SI-2015/Q2
(i)
M1

(ii)

Use of cos(A B) formula with A = 60o, B = 45o OR A = 45o, B = 30o


or 2 cos2 15o 1 etc.

3 1

A1

Exact trig.values used (visibly) to gain cos 15o =

M1

Similar method OR sin = 1 cos 2 (as 15o is acute, no requirement to justify +vesq.rt.)

A1

sin 15o =

M1
A1
A1

Use of cos(A + B) formula and double-angle formulae OR de MoivresThm. (etc.)


cos3 4cos3 3cos
Justifying/noting that x = cos is thus a root of 4 x 3 3 x cos 3 0

3 1
2 2

2 2

legitimately (Given Answer)

(however legitimately obtained)

M1

For serious attempt to factorise 4 x 3 c 3 3 x c as linear quadratic factors

A1

or via Vietas Theorem (roots/coefficients)


( x c) 4 x 2 cx c 2 3

M1


Solving 4 x 4cx 4c 3 0 FT their quadratic factor
Remaining roots are x = c c 4c 3
2

1
2

M1

Use of s 1 c 2 to simplify sq.rt. term

A1

x=

1
2

cos

3 sin

(iii)

M1

1 3 3
2
y y
0
2
2
2

A1

1
4
2

M1

cos3 =

A1

= 15o
1
y cos ,
2

M1
A1
A1
A1

1
y 3

y
0
2

2
= cos 45o
2
1
2

y = 2 cos 15o =

cos

cos

3 sin ,

1
2

3 sin with their

3 1
2

3 sin 15 o cos15 o =

3 1
2

3 sin 15 cos15 = 2
o

SI-2015/Q3

1
2

(b a)

P
1
2

(b a)

B1
M1
A1

For correct lengths in smaller


By similar s (OR trig.ORcoord.geom.)
PQ 12 (b a)
b(b a)
1
PQ =
b
ba
2 (b a )

M1

so a guard at a corner can see 2(b + PQ)

A1

4b 2
4ba
=
(might be given as all but
or as a
ba
ba
fraction of the perimeter)

Lengths
M1

1
2

A1

1
2

a and 12 (b a) in smaller

By similar s (OR trig.ORcoord.geom.)


1
1
b
a
b(b a )
2
1 2
PQ =
PQ 2 (b a)
2a

M1

so a guard at a midpoint can see b + 2PQ

A1

b2
b( 4a b)
(might be given as all but
or as a
a
a

fraction of the perimeter)

Lengths
P

M1
A1

M1
A1

1
2

a and 12 (b a) in smaller

By similar s (OR trig. OR coord.geom.)


1
a
PQ
ba
1 2
PQ =
b
ba
2 (b a )
so a guard at a midpoint can see 4b 2PQ
2ba
2b(2b 3a )
=
(might be given as all but
or as a
ba
ba
fraction of the perimeter)

B1

Recognition that b = 3a is the case when guard at M / C equally preferable


(P at corner in the two M cases)

4b 2 b 2
b2
3a b

b a a a(b a)
Correct conclusion: Guard stands at C for b< 3a and at M for b> 3a

M1A1 Relevant algebra for comparison of one case


A1

4b 2 2b(2b 3a )
2ba
3a b

ba
ba
(b a )(b a )
Correct conclusion: Guard stands at C for b< 3a and at M for b> 3a

M1A1 Relevant algebra


A1

Overall, I am anticipating that most attempts will do the Corner scenario and one of the Middle scenarios. This
will allow for a maximum of 12 = 5 (for the Corner work) + 4 (for the Middle work) + 3 (for the comparison).
In this circumstance, it wont generally be suitable to give the B1 for the b = 3a observation.

SI-2015/Q4
M1

When P is at (x,

1
4

x 2 ) ... and makes an angle of with the positive x-axis

A1

... the lower end, Q, is at x b cos ,

M1

Also, y 14 x 2

A1

x = 2 tan

dy 1
x = tan
dx 2

i.e. P = 2 tan , tan 2

1
4

x 2 b sin

A1A1 so thatQ = 2 tan b cos , tan b sin


2

obtained legitimately (Given Answer)

2 tan
cos

M1A1

When x = 0, 2 tan b cos b

M1A1

Substg. into y-coordinate yA = tan 2 2 tan

sin
tan 2
cos

M1A1

Eqn. of line AP is y x tan tan 2

M1A1

Area between curve and line is

B1

Correct limits (0, 2tan )

A1A1

1
12

A1A1

2
3

tan 3 2 tan 3 2 tan 3

A1

2
3

tan 3 obtained legitimately (Given Answer)

x 3 12 x 2 tan x tan 2

1
4

x 2 x tan tan 2

dx

(Any 2 correct terms; all 3)


(Any 2 correct terms; all 3 FT)

ALTERNATIVE
O

A
M1 A1

for obtaining the conversion factor bcos = 2 tan or tan2 = 12 b sin

M1 A1

for distances OB = BC (= 12 b cos ) and so PC = OA = tan2

M1 A1

giving OABCPB

A1

Area is

B1
A1 A1

Correct limits (0, 2 tan ) used


Correct integration; correct Given Answer

ALTERNATIVE

1
4

x 2 dx

Translate whole thing up by tan2 and calculate

b cos
1
4
0

x 2 tan 2 dx

SI-2015/Q5
(i)

M1A1

(t 1) x 3
f(x) =

x 1

A1

M1

Differentiating by use of Quotient RuleOR taking logs.anddiffg. implicitly)


d x
for
2 2 x. ln 2 seen at any stage
dx

B1
A1
A1

2x
x

x .2 x . ln 2 2 x
dy
=
dx
x2
1

TP at
, e ln 2
ln 2

(y-coordinate not required)

B1

Jusitfying that the TP is a minimum

G1
G1

Generally correct -shape


Asymptotic to y-axis
and TP in FT correct position

(ii)

M1
A1
B1
M1A1
A1

Let u 2 1 x 2 2 xt
2u du = 2xdt
t: (1, 1) u: ( | 1 + x |, | 1 x | ) Correct limits seen at any stage
1
Full substn. attempt; correct g(x) =
1 . du
x
1
g(x) = 1 x 1 x
n. may be done directly, but be strict on the limits
x
2
x 1
x

org(x) = 2 1 x 1

2
x 1
x
(Must have completely correct three intervals: x< 1, 1 x 1 , x> 1)

M1
A1 A1

Graph split into two or three regions


Reciprocal graphs on LHS & RHS (must be asymptotic to x-axis)

A1

Horizontal line for middle segment

(Allow even if they approach y-axis also)

SI-2015/Q6

Let P, Q, R and S be the midpoints of sides (as shown)


Then
M1A1
p= 12 a 12 b , q = 12 b 12 a ,

r= 12 a 12 b , s = 12 b 12 a

and
M1A1
A1
A1

a a
QR PS 12 b b
PQ SR

1
2

so that PQSR is a //gm.


(opposite sides // and equal)

M1
A1

PQ QR PQ QS
=

1
4

1
2

a a 12 b b

a b a b a b a b

for use of the scalar product


Do not accept a b etc.

M1

Use of perpendicularity of OA, OB and OA, OB


= 14 a b a b

M1
A1

AOB = AOB = 180o ; and cos(180o ) = cos


= 0 since a b ab cos and a b a b cos
and we are given that a = b and a = b
so that PQRS is a rectangle (adjacent sides perpendicular)

A1

( a ) 2 2a a

B1

PQ 2 SR 2 PQ PQ =

B1

QR 2 PS 2 =

M1

Since a = b, a = b and a a aa cos 90 o , b ab bb cos 90 o

A1

it follows that PQRS is a square (adjacent sides equal)

1
4

1
4

(b ) 2 2b b

( a ) 2 2aa cos 90 o

M1A1

Area PQRS =

M1

... which is maximal when cos 90 o 1

A1

i.e. when 90 o

1
4

SI-2015/Q7
M1

f (x) = 6ax 18x2


= 6x(a 3x)
= 0 for x = 0 and

A1A1

f(0) = 0

A1A1

f( 13 a) = 19 a3

(Min. TP)

A1

x = 13 a

(Max. TP)

since f(x) is a negative cubic

(f(0) = 0 and the TPs may be shown on a sketch award the marks here if necessary)

Evaluating at the endpoints


f( 13 ) = 19 (3a + 2); f(1) = 3a 6

M1
A1A1

1
9

M1

(3a + 2) 19 a3 a3 3a 2 0

(a + 1)2(a 2) 0
and since a 0, a 2

M1
A1

1
9

M1

a3 3a 6 a3 27a + 54 0

(a 3)2(a + 6) 0
which holds for all a 0

M1
A1

1
9

M1

(3a + 2) 3a 6 3a + 2 27a 54
8(3a 7) 0
a 73

A1

(which, actually, affects nothing, but working should appear)

Thus
B1B1B1

19 (3a 2) 0 a 2

M(a) = 19 a 3
2a3
3a 6
a3

(Ignore non-unique allocation of endpoints)

(Do not award marks for correct answers unsupported or from incorrect working)

SI-2015/Q8
(i)
M1
M1
A1

S = 1 + 2 + 3 + + (n 2) + (n 1) + n
S = n + (n 1) + (n 2) + + 3 + 2 + 1
2S = n (n + 1)
S = 12 n(n + 1) obtained legitimately (Given Answer)

Method
Adding

(Allow alternatives using induction or the Method of Differences, for instance, but NOT by stating
that it is an AP and just quoting a formula; ditto -number formula)

(ii)
M1A1
E1

(N m)k + mk (k odd)
k
k
k k 1
m N m k m k
= N k mN k 1 m 2 N k 2 ...
1
2
k 1
which is clearly divisible by N (since each term has a factor of N)
(Allow alternatives using induction, for instance)

Let S = 1k + 2k + + nk

an odd no. of terms

M1

= 0k + 1k + 2k + + nk

M1

= (n 0) k 0 k ( n 1) k 1k ... ( 12 n 12 ) k ( 12 n 12 ) k

an even no. of terms

(no need to demonstrate final pairing but must explain fully the pairing up or the single extra nk term)

E1

and, by (ii), each term is divisible by n.

For

S = 1k + 2k + + nk

an even no. of terms

M1

= 0k + 1k + 2k + + nk

M1

= (n 0) k 0 k (n 1) k 1k ... ( 12 n 1) k ( 12 n 1) k 12 n

an odd no. of terms

(no need to demonstrate final pairing but must explain the pairing and note the separate, single term)

E1

and, by (ii), each paired term is divisible by n


and the final single term is divisible by 12 n required result

M1
A1
E1

By the above result for n even, so that (n + 1) is odd


(n + 1) | 1k + 2k + + nk + (n + 1)k
(n + 1) |S + (n + 1)k (n + 1) |S

M1
A1

By the above result for n odd, so that (n + 1) is even


k
k
k
k
1
2 (n + 1) | 1 + 2 + + n + (n + 1)

E1

1
2

(n + 1) | S + (n + 1)k

1
2

(n + 1) |S (as

1
2

(n + 1) is an integer)

E1

Since hcf(n, n + 1) = 1 hcf( 12 n, n + 1) = 1 for n even

E1

and hcf(n,
So it follows that

1
2

1
2

(n + 1)) = 1 for n odd

n(n + 1) | S for all positive integers n

SI/15/Q9
M1

Time taken to land (at the level of the projection) (from y = utsin
is t

A1
M1
A1
M1A1
A1

Bullet fired at time t 0 t


lands at time
6

2u

TL t sin t
g
3

1
dTL
2 u

1
cos t = k cos t
k
g
dt
3

= 0 when k cos t
3

2u 2 sin cos
(from y = utsin
g

Horizontal range is R

A1

RL

M1A1

d 2TL
22 u

sin t 0 maximum distance


2
g
dt
3

M1A1

0t

A1
M1
A1
M1A1

gt2 , y = 0, t 0)

2u sin
(may be implicit)
g

M1A1

M1

1
2

2u 2
k 1 k 2
g

1
2

gt2 with above time)

obtained legitimately (Given Answer)

1
3

in k cos t k
6
2
2
3

dTL
0 throughout the guns firing
dt
and TL is a (strictly) decreasing function.
Then TL max. occurs at t = 0
If k <

1
2

then

i.e.
and RL

2u 1
3 u2 3

g 2 2
2g

SI-2015/Q10
B1

Speed of rain relative to bus is vcos u (or u vcos if negative)

M1A1

When u = 0, A hvcos + avsin (width of bus and time units may be included as factors)

E1

When vcos u > 0, rain hitting top of bus is the same, and rain hits back of bus
as before, but with vcos u instead of vcos
When vcos u < 0, rain hitting top of bus is the same, and rain hits front of bus
as before, but with u vcos instead of vcos
Together, A h |vcos u| + avsin
Fully justified (Given Answer)

E1
A1

M1
A1
M1

1
so we need to minimise
u
av sin h | v cos u |
J

(Ignore additional constant-of-proportionality factors)


u
u

Journey time

For vcos u > 0,


if w vcos , we minimise J

av sin hv cos

h
u
u

E1
E1

and this decreases as u increases


and this is done by choosing u as large as possible; i.e. u = w

M1

For u vcos > 0,

av sin hv cos

h
u
u
and this decreases as u increases if a sin > hcos
so we again choose u as large as possible; i.e. u = w

we minimise J
E1
E1

[Note: minimisation may be justified by calculus in either case or both.]

M1
A1

If a sin < hcos , then J increases with u when u exceeds vcos


so we choose u = vcos in this case

M1A1

M1
A1

If a sin = hcos then J is independent of u, so we may as well take u = w

av sin hv cos

h
u
u
Which always decreases as u increases, so take u = w again
Replacing by 180o gives J

SI-2015/Q11
(i)

B1

O1: F = F1
O2: F = F2

O1
(Both, with reason)

O2

(ii)

B1

Resg. ||plane (for C1):

F1 R W1 sin

B1

Resg.r. plane (for C1):

R1 F W1 cos

B1

Resg.||plane (for C2):

F2 R W2 sin

B1

Resg.r. plane (for C2):

R2 F W2 cos

Max 4 marks to be given for four independent statements (though only 3 are required).
One or other of
Resg.||plane (for system) : F1 F2 W1 W2 sin
Resg.r. plane (for system) : R1 R2 W1 W2 cos
may also appear instead of one or more of the above.
(F1 and F2 may or may not appear in these statements as F, but should do so below)

FR FR
using and

W1
W2

M1A1

Equating for sin :

M1A1

W W2
R
Re-arranging for F in terms of R: F 1
W1 W2

M1

Use of the Friction Law, F R

A1

W1 W2

W1 W2

obtained legitimately (Given Answer)

M1A1

M1A1

(e.g.) tan

W W2

F F 1
W1 W2
W W2

F
=
using R 1
W

W
R1 F
2
1

Substg. for R

2W1

F
W
W

2
= 1
R1 F1

A1
M1A1

FR
R1 F

Substg. for R1 (correct inequality) using Friction Law F1 1 R1 R1

F1

2W1

F
W1 W2

F1
F1

M1

A1

Tidying-up algebra

2W1

F
W1 W2

=
1 1

F
1

tan

2 1W1
obtained legitimately (Given Answer)
1 1 W1 W2

SI-2015/Q12
r

(i)

M1A1

n 1 3
P(exactly r out of n need surgery) =
r 4 4

nr

(A binomial prob. term; correct)

(ii)

M1
B1B1A1

nr

M1

e 8
r!

8n
1 3

n r (n r ) ! 4 4

M1

e 8
r!

8n
3n r

Attempting to deal with the powers of 3 and 4

4n
n r (n r ) !

A1

e 8
=
r!

M1

e 8 2 r
r!

6n r
Splitting off the extra powers of 2 ready to

n r (n r ) !

M1

e 8 2 r
r!

A1

(iii)

nr

e 8 8 n
n!
1 3

Attempt at sum of appropriate product terms

r ! (n r ) ! 4 4
nr n!
Limits
All internal terms correct; allow nCr for the A mark

P(S = r) =

Factoring out these two terms

2 n 3n r
Correctly

n r (n r ) !

m0

e 8 2 r
e6
r!

6m
adjust the lower limit (i.e. using m = n r)
m!

i.e.

e 2 2 r
r!

A1

which is Poisson with mean 2

M1

P(M = 8 |M + T = 12)

Identifying correct conditional probability outcome

A1A1A1

e 2 28 e 2 2 4

8!
4!
=
e 4 412
12 !

One A mark for each correct term (& no extras for 3rd A mark)

A1A1

212 12 !
412 8 ! 4 !

(Give B1 for noting this without the working)

Powers of e cancelled; factorials in correct part of the fraction


(unsimplified is okay at this stage)

A1

495
4096

SI-2015/Q13
Reminder
A : the 1st6 arises on the nth throw
B : at least one 5 arises before the 1st6
C : at least one 4 arises before the 1st6
D : exactly one 5 arises before the 1st6
E : exactly one 4 arises before the 1st6

56 n 1 16

(i)

M1A1

P(A) =

(ii)

M1A1

By symmetry (either a 5 or a 6 arises before the other), P(B) =

(iii)

M1

The first 4s, 5s, 6s can arise in the orders 456, 465, 546, 564, 645, 654

A1

P(BC) =

1
3

1
2

(i.e. by symmetry but with three pairs)

(iv)

M1A1A1

P(D) =

16 16 16 64 16 16 64 2 16 ...
2

1
M1 for infinite series with 1stterm ; A1 for 2nd term ; A1 for 3rd term and following pattern

M1

361 1 2 23 3 23 2 ...

For factorisation and an infinite series

M1

361 1 23 2

Use of the given series result

A1

1
4

(v)

M1

P(DE) = P(D) + P(E) P(DE)

Stated or used

B1

P(E) = P(D) = answer to (iv)

Stated or used anywhere

M1A1A1

P(DE) =

62 16 16 63 62 16 16 63 2 62 16 16 ...
3

M1 for infinite series with 1 term ; A1 for 2nd term ; A1 for 3rd term and following pattern
st

M1

1081 1 3 12 6 12 2 ...

For factorisation and an infinite series

M1

1081 1 12 3

Use of the given series result

A1

P(DE) =

1
2

272

23
54

Question1
(i)

ln 1

For

Therefore

If
0,
Therefore

ln 1
ln 1

Therefore

ln 1

So,

ln 1

So,

ln 1

B1

ln

ln

ln

ln

M1

M1

ln 1

A1

ln 1

Therefore

If
0,
Therefore

So,

Therefore

B1

M1

ln 1

0for0

ln 1
ln 1

1.

0
isnegativefor0

ln 1

0forall
1

ln 1
1

1,

ln 1

B1

1.
1.

B1

ln

A1

ln

2 ln

ln

M1M1
A1

So,
ln 1
As ,ln
Therefore,
1

1
1

ln 2

ln

ln
1

ln

M1A1

1 0

B1

A1

B1

ln

ln

A1

0forallpositive .

For0

0
ispositiveforallpositive .

ln

0for

Therefore,
1

(ii)

B1

M1

ln 1

ln 1

0,

ln 2

Question1
Notethatthestatementofthequestionrequirestheuseofaparticularmethodinbothparts.
(i)
B1
M1
A1
B1

Correctdifferentiationoftheexpression.
Considerationofthesignofthederivativeforpositivevaluesof .
Deductionthatthederivativeispositiveforallpositivevaluesof .
Clearexplanationthat
ln 1
ispositiveforallpositive .
Notethatanswerisgiveninthequestion.

B1

Useof

andsummation.

M1 Manipulationoflogarithmicexpressiontoformdifference.
M1 Attempttosimplifythesum(somepairscancelledoutwithinsum).
A1 Clearexplanationofresult.
Notethatanswerisgiveninthequestion.

(ii)
B1 Correctdifferentiationoftheexpression.
M1 Considerationofthesignofthederivativefor0
1.
A1 Deductionthatthederivativeisnegativeforthisrangeofvalues.
B1 Deductionthat
ln 1
isnegativeforthisrangeofvalues.
B1 Useof
andsummation.
M1 Expressionwithinlogarithmasasinglefractionandnumeratorsimplified.
M1 Logarithmsplittochangeatleastoneproducttoasumoflogarithmsoronequotientasa
differenceoflogarithms.
A1 Completesplitoflogarithmtorequiredform.
M1 Useofdifferencestosimplifysum.
A1 ln 2correct.
B1 Correctlydealingwithlimitas .
Notethatanswerswhichuseastheupperlimitonthesumfromthebeginningmust
haveclearjustificationofthelimit.Thosebeginningwith astheupperlimitmusthave
ln
ln
1 correctinsimplifiedsum.
A1 Inclusionof
1tothesumtoreachthefinalanswer.
Notethatanswerisgiveninthequestion.

Question2

sin
sin

B1
M1A1

sin
sin 3

3
3

M1
2 sin cos

(or

M1M1

A1

B1

3
3

4 sin

cos 2

4 sin

2 1

Therefore
3
1
1
3

3
3
So trisectstheangle

sin

,so

B1

sin 3

sin
sin cos 2
cos sin 2

sin
cos
sin
1 2 sin
sin
3 4 sin

B1B1

2 sin

M1M1
A1

B1
M1A1

M1M1
A1

Question2
B1
M1
A1
B1
M1
M1
M1
A1

B1
B1
B1
M1
M1
A1

B1
M1
A1
M1

(maybeimpliedbylaterworking).
Expressionfor
Applicationofthesinerule.
Correctstatement.
Doesnotneedtobestatedaslongasimpliedinworking.
Useofsin
formula.
Useofdoubleangleformulaforsin.
Useofdoubleangleformulaforcos.
Simplificationofexpression.
Notethatanswerisgiveninthequestion.

Identificationofthisrelationshipbetweendistances.(just
Correctexpressionsubstitutedforthelengthof .
Correctexpressionsubstitutedforthelengthof .
Useofdoubleangleformulaforcos.
Simplificationofexpressionobtained.
Correctexpressionindependentof .

Identificationofarightangledtriangletocalculatesin
.
Deductionthatoneofthelengthsinsineofthisangleisequalto
Valueoftheangle(Degreesorradiansarebothacceptable).
Obtaining
2

M1 Useof

A1 Expressiontoshowthat

andconclusionstated.

issufficient)

Question3

isthenumberoftrianglesthatcanbemadeusingarodoflength8and
twoother,shorterrods.
Ifthemiddlelengthrodhaslength7thentheotherrodcanbe1,2,3,4,5or6.
Ifthemiddlelengthrodhaslength6thentheotherrodcanbe2,3,4or5.
Ifthemiddlelengthrodhaslength5thentheotherrodcanbe3or4.
2 4 6.

Assumethatthelongestofthethreerodshaslength7:
Ifthemiddlelengthrodhaslength6thentheotherrodcanbe1,2,3,4or5.
Ifthemiddlelengthrodhaslength5thentheotherrodcanbe2,3or4.
Ifthemiddlelengthrodhaslength4thentheotherrodmustbe3.
Therefore
1 3 5.
1 2 3 4 5 6.

2 4 2
1
1 2 3 2
1

3.(Thepossibilitiesare 1, 2, 3 , 1, 3, 4 and 2, 3, 4 .)
Substituting

2intotheequationgives

Thereforetheformulaiscorrectinthecase
Assumethattheformulaiscorrectinthecase

2 2

1 4

3.

2.

3
1 12
4
statementoftheformulawhere
Therefore,byinduction,

2
1 4

Therefore
1 4

5 .(Or

M1
4

1 ,whichisa

1.
1 4

M1
A1

M1
M1

M1
A1
A1

B1
B1

B1

M1

1 4

M1

B1

M1

1
1

M1

A1

1 .

M1A1

1 .
1 4

1 )

A1

Question3
M1
M1
M1
A1
M1
M1
M1
A1
A1
B1
B1
B1
B1
M1
M1
M1
A1
M1
A1
A1

.
Appreciationofthemeaningof
Identifythenumberofpossibilitiesforthelengthofthethirdrodinonecase.
Identifythesetofpossiblecasesandfindnumbersofpossibilitiesforeach.
Clearexplanationoftheresult.
Notethatanswerisgiveninthequestion.
Anattempttoworkout
.
Correctcalculationforanyonedefinedcase.
Identificationofacompletesetofcases.
Correctvaluefor
.
Correctdeductionofexpressionfor
.
Correctexpression.Nojustificationisneededforthismark.
Correctexpression.Nojustificationisneededforthismark.
Correctjustificationthat
3.Requiressightofpossibilitiesorotherjustification.
Evidenceofcheckingabasecase.(Acceptconfirmationthat
1gives
0here.)
Applicationofthepreviouslydeducedresult.
Substitutionofformulafor
andtheformulaforthesum.
Takingcommonfactortogiveasingleproduct.
Rearrangementtoshowthatitisastatementoftherequiredformulawhen
1
andconclusionstated.
Useofresultfromstartofquestion.
Correctsummationof2 4 2
1 .
Correctformulareached(anyequivalentexpressionisacceptable).

Question4
(i)

B1
B1

(ii)

,sostationarypointswhen

1.
B1
B1
M1
A1
A1

B1
B1
B1
B1

(iii)
B1
B1
B1
B1

B1
B1
B1
B1
B1


Question4
Penaliseadditionalsectionstographs(verticaltranslationsby
providingthatthecorrectsectionispresentinlaterparts.

)onlyonthefirstoccasion

B1
B1

Correctshape.

B1
B1
M1
A1
A1

B1
B1
B1

Rotationalsymmetryaboutthepoint 0,0 .
Correctshape.
Differentiationtofindstationarypoints.
.( coordinates)
Correctstationarypoints 1,
Correct coordinates.

Rotationalsymmetryaboutthepoint 0, .
Correctshape.
Stationarypointshavesame coordinatesaspreviousgraph.(Followthroughincorrect
stationarypointsinpreviousgraphifconsistenthere).
Correctcoordinatesforstationarypoints 1,
arctan

Correctasymptotes
1.
axisasanasymptote.
Middlesectioncorrectshape.
Outsidesectionscorrectshape.

Sectionfor 1
1correctshape.
1
.

B1

B1
B1
B1
B1

B1
B1
B1
B1
B1

Asymptotes

and

shown.

Sectionfor
Sectionfor
0, .

1correctwithasymptote
1correctwithasymptote

2 .
0orarotationof

1sectionabout

Question5
(i)

tan

tan arctan

arctan

M1

tan

tan

,whichsimplifiesto

tan

M1

M1A1

Hence,byinductiontan

Supposethatitisnottruethat

arctan

Clearly,

A1

B1

.
arctan

forallvaluesof .
arctan

Thenthereisasmallestpositivevalue, suchthat

Since

arctan

andtan

However,

Therefore

arctan
arctan

M1M1
A1

tan 2

M1A1

So,

Whichsimplifiesto2 tan
tan
2 tan
2
Since mustbeacute,tan

B1

arctan

Therefore
lim

tan
1 4
1
0
cannotequal 2 .

arctan 1

M1A1
A1
B1

A1

,sothisisnotpossible.

.
arctan

,but

(ii)

B1

Assumethattan

1.

,sotheformulaiscorrectfor

M1A1

Question5
Confirmationthattheformulaiscorrectfor
1.
Expressionof
intermsof .
Useoftan
formula.
Simplificationoffraction.
Expressionof
toshowthatitmatchesresult.
Conclusionstated.

Confirmationfor
1.
Observationthat
0
Evidenceofunderstandingthatsuccessivevaluesof withthesamevalueoftan must
differby .
A1 Evidenceofunderstandingthat
cannotbesufficientlylargefor tobeofthe
formarctan if
is.
A1 Clearjustification.

M1 Identificationoftherelevantsidesofthetriangle(diagramissufficient).
A1 Correctexpressionfortan 2 .
B1 Useofdoubleangleformula.
M1 Rearrangementtoremovefractions.
A1 Correctquadraticreached.
A1 Quadraticfactorised.
B1 Irrelevantcaseeliminated(mustbejustified).
M1 Sumexpressedaslimitof
A1 Correctvaluejustified.
Notethatanswerisgiveninthequestion.

B1
M1
M1
M1
A1
A1

B1
M1
M1

Question6
(i)

sec

cos cos
2

sec

cos

cos

sin

Therefore,sec

(ii)

sin

sin

M1
M1A1

tan

M1A1

1
sin

B1
sin
sin

Therefore,

sin
sin

sin
sin

sin

B1

sin

sin

So,2

M1

A1

,andapplyingtheresultfrompart(i):
tan

Consider

tan

So,

Therefore,

sin

sin
sin
2

Andso,

Therefore,

2.

.Makingthesubstitution

B1

sin

M1A1

sin

sec

sec

tan
tan

sec

B1
M1
A1
B1

B1

1
sin

2 tan

tan

sin

2 sin cos

(iii)

Limits:

B1

sin

Therefore,

sin sin

cos

B1

sec

cos

B1

Question6
B1
B1

Expressionofsec intermsofanyothertrigonometricfunctions.
Correctuseofaformulasuchasthatforcos
toobtainexpressionwith
trigonometricfunctionsof .
M1 Expandingthesquaredbrackets.
M1 Useofsin 2 sin cos andsin
cos
1
A1

M1
A1
B1

B1
M1
A1
B1
B1

M1
A1
B1
M1
A1
B1
B1

Fullyjustifiedanswer.
Notethatanswerisgiveninthequestion.

.
Anymultipleoftan
Correctanswer
Dealswithchangeoflimitscorrectly.AND
Correctlydealswithchangetointegralwithrespectto .
Notethatboththesestepsneedtobeseenthecorrectresultreachedwithoutevidence
ofthesestepsshouldnotscorethismark.
Useofsin
sin (maybejustseenwithinworking)
Groupingsimilarintegrals.
Fullyjustifiedanswer.
Notethatanswerisgiveninthequestion.
Evaluationoftheintegralfrom(i)withtheappropriatelimits.
Useofresultfrom(ii)toevaluaterequiredintegral.

Attempttomakethesubstitution.
Substitutionallcompletedcorrectly.
Rearrangetogivesomethingthatcanrepresenttherequiredintegralononeside.
1 tan withinintegral.
Useofsec
Correctevaluationofthisintegral.
Correctuseofresultfrompart(i).
Correctapplicationofresultdeducedearliertoreachfinalanswer.

Question7
(i)

(ii)

Mostlikelyexamples:

and

If
thentherecannotbetwopointsonthecirclethatareadistanceof2
apartandanytwodiametricallyoppositepointson mustbeadistanceof2
apart.
If
thenthecirclemustbethesameas ,sothereisnotexactly2points
ofintersection.

Thedistancesofthecentreof fromthecentresof and are

and
.
Ifthe coordinateofthecentreof is ,thenthe coordinateisgivenby
and

Therefore,
4

andso

Therefore,the coordinateofthecentreof satisfies

16
16
16
16
16
16

M1A1
B1
B1B1
M1

B1

So,

Therefore,
8
16
4
4
4
4

16

B1
0

8
4
2
4

B1

So2

B1

M1A1

and

M1M1
A1

M1M1

M1
M1

A1

Question7
M1
M1
A1
B1
B1

M1
A1
B1
B1
B1
M1
M1
A1

B1

B1

M1
M1
M1
M1
A1

.
Calculationthatthedistancebetweenthecentresofthecirclesmustbe
Anexamplewhichshowsthatitispossibleforatleastonevalueof .
Exampleshowingthatitispossibleforall
.
Statementthatthetwointersectionpointsmustbeadistance2 apart.
Explanationthatinthecase
itwouldhavetobethesamecircle.

Thelinejoiningthecentreof (or )andtheradiitoapointofintersectionformaright


angledtriangleineachcase.(onecase)
Useofthistofindthedistancebetweencentresofcircles.
Applyingthesametotheothercircle.
Expressionrelatingthecoordinatesandradiiobtainedfromconsidering .
Expressionrelatingthecoordinatesandradiiobtainedfromconsidering .
Eliminationof fromtheequations.
Eitherexpansionofsquaredtermsorrearrangementtoapplydifferenceoftwosquares.
Expressionfor reached.
Notethatanswerisgiveninthequestion.

Substitutiontofindexpressionfor coordinate.
Notethatanyexpressionfor intermsof , , and issufficient,butitmustbe
.
expressedas
,not
Observationthat mustbepositive.
Alternativemarkschemeforthismayberequiredoncesomesolutionsseen.
ontheleft.
Attempttorearrangetheinequalitytoget16
Reachapointsymmetricin and .
Reachacombinationofsquaredterms.
Applydifferenceoftwosquarestosimplify.
Reachtherequiredinequality.
Notethatanswerisgiveninthequestion.

Question8
(i)

Let bethevectorfromthecentreof to .
Usingsimilartriangles,thevectorfromthecentreof to is

Therefore

fromthecentreof tothecentreof

So

Thepositionvectorof is

(ii)

M1A1

,sincethesearebothexpressionsforthevector
.

A1

Thepositionvectorsof and willbe

M1A1

and

B1

Therefore,

Similarly,

Sincetheyaremultiplesofeachotherthepoints , and mustlieonthe


samestraightline.

lieshalfwaybetween and if

Therefore

So,
Whichsimplifiesto

M1A1
M1A1

(iii)

M1

M1A1
M1A1
B1

B1
M1

M1A1

Question8
M1
A1
M1
A1
M1
A1

B1
M1
A1
M1
A1
M1
A1
M1
A1
B1

B1
M1
M1
A1

Identificationofsimilartriangleswithinthediagram.
Relationshipbetweenthetwovectorsto .
Equatingtwoexpressionsforthevectorbetweenthecentresofthecircles.
Correctsimplifiedexpression.
Calculationofvectorfromcentreofonecircleto .
Correctpositionvectorfor .
Notethatanswerisgiveninthequestion.

Identifyingthecorrectvectorsforthefocioftheotherpairsofcircles.
Expressionforvectorbetweenanytwoofthefoci.
Termsgroupedbyvector.
Simplificationofgroupedterms.
Extractionofcommonfactor.
Expressionforavectorbetweenadifferentpairoffoci.
Awardmarksassameschemeforpreviousexample,butawardallfourmarksforthe
correctanswerwrittendownasitcanbeobtainedbyrotating1,2and3intheprevious
answer.
Statementthattheylieonastraightline.

Statementthatthetwovectorsmustbeequal.
Reductiontostatementinvolvingonly terms.
Attempttosimplifyexpressionobtained(ifnecessary).
Anysimplifiedform.

Question9
(i)

Takingmomentsabout :
3
sin 30

5
sin 30

5
cos 30 sin
cos sin 30
3
cos 30 sin
cos sin 30
1
1
3
3
sin
cos
3
sin
cos
5
2
2
2
2
Therefore
cos
43 sin
Either
Usesin
cos
1andjustifychoiceofpositivesquareroot.
Or
Drawrightangledtrianglesuchthattan
andcalculatethelengthofthe

M1A1
M1A1
B1
M1
A1
A1

M1

hypotenuse.

(ii)

sin

A1

Let betheverticaldistanceof below .


Let betheverticaldistanceof below .

sin

M1M1
A1

sin

M1A1

If isthecentreofmassofthetriangle:

M1A1

Conservationofenergy:
4
8 . 2 forcompleterevolutions.

M1A1

Therefore

A1

Question9
M1
A1
M1
A1
B1
M1
A1
A1
M1
A1

M1
M1
A1
M1
A1
M1
A1
M1
A1
A1

Attempttofindthemomentof about .
Correctexpressionformoment(sin 30
maybereplacedbycos 60
).
Attempttofindthemomentof about .
Correctexpressionformoment(sin 30
maybereplacedbycos 60
).
Correctstatementthatthesemustbeequal.
Useofsin
orcos
formulae.
Correctvaluesusedforsin 30andcos 30.
Correctlysimplified.
Useofacorrectmethodtofindthevalueofsin .
Fullyjustifiedsolution.Ifusingrightangledtrianglemethodthenchoiceofpositiverootnot
needed,ifchoiceofpositiverootnotgivenwhenapplyingsin
cos
1method
thenM1A0shouldbeawarded.
Notethatanswerisgiveninthequestion.

Attempttofind .
Correctlydealwithsineorcosineterm.
Correctvalue.
Attempttofind .
Correctvalue.
Combinetwovaluestoobtaindistanceofcentreofmassfrom .
Correctvalue
Applyconservationofenergy.
Correctinequality.
Correctminimumvalue.

Question9Alternativepart(i)
(i)

Let bethecentreofmassofthetriangleandletthedistance
Takingmomentsabout :
5
cos
3
cos
Therefore5
3
,so
.
mustlieon and
30
.
cos

sin 30
sin 30 cos

cos 30 sin

43 sin andsocos

,andso(since isacute)sin

48 sin

Takingmoments.
Correctequation.
Correctrelationshipbetween and .
Identificationthat lieson andcalculationof
.
Useofsineofidentifiedangle.
Useofsin
formula.
Directrelationshipbetweensin andcos .
Rearrangementandsquaringbothsides.
cos
1.
Applyingsin
Finalanswer(choiceofpositiverootmustbeexplained).
Notethatanswerisgiveninthequestion.

M1A1
A1
B1
M1

A1

M1
A1
A1
B1
M1
M1
A1
M1
M1
A1

M1

Thereforecos
sin

cos

be .

M1
M1A1

Question10

Ifthelengthofstringfromtheholeatanymomentis ,then

Thedistance, ,fromthepointbeneaththeholesatisfies,
2

Therefore

,and

Therefore,thespeedoftheparticleis cosec .

sin

Therefore

Theaccelerationis

Since

Horizontally:

A1

cosec cot

M1A1

A1
M1

sec ,theaccelerationcanbewrittenas

cot

M1A1

cot

,so

cot

andsotan

M1M1
A1

cosec

Theparticlewillleavethefloorwhen cos
cot

M1

cot

M1

cot

B1
M1A1

,socos

sin

cosec

Acceleration:

cosec

B1

M1A1
M1A1

Question10
B1
B1
M1
A1
M1
A1

Aninterpretationof intermsofothervariables(includinganynewlydefinedones).
Anyvalidrelationshipbetweenthevariables.
Differentiationtofindhorizontalvelocity.
Correctdifferentiation.
Attempttoeliminateanyintroducedvariables.
Correctresult.
Answerswhichmakeclearreferencetothespeedoftheparticleinthedirectionofthe
stringbeingV.

M1 Differentiationofspeedfoundinfirstpart.
A1 Correctanswer.
M1 Attempttodifferentiatetofindanexpressionfor .
A1
M1
M1
A1

M1
M1
A1

M1
A1
M1
A1

Correctanswer.
Substitutiontofindexpressionforacceleration.
Relationshipbetweenrequiredvariablesandanyextravariablesidentified.
Substitutiontogiveanswerintermsofcorrectvariables.

Horizontalcomponentoftension.
ApplicationofNewtonssecondlaw.
Correctanswer.

Verticalcomponentoftensionfound.
Identificationthatparticleleavesgroundwhentensionisequaltothemass.
Substitutionoftheirvaluefor .
Rearrangementtogiverequiredresult.
Notethatanswerisgiveninthequestion.

Question11
(i)

(ii)

cos , sin
Differentiating:
sin
, cos

Since ismovingwithvelocity andisatthepoint , 0 attime ,


:
sin ,
cos .
Velocityof is

Initialmomentumwas (horizontally).
Horizontalvelocityof willbethesameasthatof ,sohorizontallythetotal
momentumisgivenby
2
sin
Therefore3
2 sin
.

Initialenergywas

Totalenergyis

Therefore
3
So

(iii)

(iv)

A1

2
2

sin

cos

M1A1

sin

cos

M1

gives
2

3
4

sin
sin

4 sin
4
sin
4
2

2 sin
sin
4

M1

sin

sin

A1

So,

0,sotherecanonlybeaninstantaneouschangeofdirectioninwhich
variesatacollision.Sincethefirstcollisionwillbewhen
0,thesecond
collisionmustbewhen
.

Sincehorizontalmomentummustbe ,
0 2 sin
.
TheKEof mustbe

,so

sin

isonly0when takesthesevaluesand ispositiveas wouldneedanon


zerovaluetosatisfy3
2 sin
if isnegative.(Therelationshipisstill
truesincecollisionsareelastic).

sin
,so

or

sin

Substituting
3
6
6

M1

M1

2
sin

B1B1
M1

A1

M1

B1
B1

B1
B1

M1A1
B1

Question11
B1
B1
M1
A1

M1
M1
A1
M1
M1
A1
M1
M1
A1

B1
B1

B1
B1
M1
A1
B1

Horizontalcomponent.
Verticalcomponent.
Differentiation.
Completejustification,includingclearexplanationthat
.
Notethatanswerisgiveninthequestion.

Statementthatmomentumwillbeconserved.
Identificationthathorizontalmomentumof and willbeequal.
Correctequationreached.
Notethatanswerisgiveninthequestion.
Statementthatenergywillbeconserved.
Useofsymmetrytoobtainenergyof (acceptanswerswhichsimplydoubletheenergyof
ratherthanstatingtheverticalvelocityinoppositedirection).
Correctrelationship.
cos
1.
Useofsin
Substitutingtheotherrelationshiptoeliminate .
Correctequationreached.
Notethatanswerisgiveninthequestion.

Correctvalueof .
Answerjustified.

Firstequationidentified.
Secondequationidentified.
Solvingsimultaneouslytofind .
Correctvaluesfor .
Justifiedanswerthat isnotalways0when takesthesevalues.

Question12
(i)

Ifatailoccursthenplayer mustalwayswinbefore canachievethe


sequencerequired.Thereforetheonlywayfor towinisifbothofthefirst
twotossesareheads.
Afterthefirsttwotossesareheadsitdoesnotmatterifmoretossesresultin
headsasthefirsttimetailsoccurs willwin.
Theprobabilitythat winsistherefore

B1
B1

(ii)

Asbefore,after ,only canwin.


Similarly,after ,only canwin.
Inallothercasesforthefirsttwotossesonly and willbeabletowin.
Theprobabilitiesfor and towinmustbeequal.

B1
B1
M1
M1

Theprobabilityofwinningis foralloftheplayers.

A1

(iii)

Ifthefirsttwotossesare then mustwin(assoonasa occurs),sothe


probabilityis1.
After :
mustwinifthenexttossisa as needstwo stowin,but willwinthe
nexttimean occurs.
Ifthenexttossis ,thenthepositionisasifthefirsttwotosseshadbeen ,
andsotheprobabilitythat winsfromthispointis .

B1
M1
M1

Therefore,

After :
Ifthenexttossis then willwinwithprobability .
Ifthenexttossis then willwinwithprobability .

M1

,andso

A1

Therefore

A1

After :
Ifthenexttossis thenplayer winsimmediately.
Ifthenexttossis then willwinwithprobability .
Therefore

M1

A1

Fromthethirdequation

Solvingthetwoequationsin and ,gives

M1A1

Theprobabilitythat winsis

B1

M1A1

Question12
B1
B1
B1

B1
B1
M1
M1
A1

B1
M1
M1
A1

M1
A1

M1
A1

M1
A1
M1
A1

Identifyingthat cannotwinonceatailhasbeentossed.
Identifyingthat mustwinoncethefirsttwotosseshavebeenheads.
Showingthecalculationtoreachtheanswer.
Notethatanswerisgiveninthequestion.

Recognisingthatthesituationisunchangedforplayer .
Recognisingthatthesamelogicappliestoplayer .
Allothercasesleadtowinsforoneoftheremainingplayers.
Recognisingthattheprobabilitiesmustbeequal.
Correctstatementoftheprobabilities.
Ifnomarkspossiblebythisschemeawardonemarkforeachprobabilitycorrectly
calculatedwithsupportingworking.Allfourcalculatedscores5marks.

Explanationthatprobabilitymustbe1.
Explanationofthecasethatthenexttossis .
Thismarkandthenextcouldbeawardedforanappropriatetreediagram.
Explanationofthecasethatthenexttossis .
Justificationoftherelationshipbetween and .
Notethatanswerisgiveninthequestion.

Considerationofonecasefollowing .
Thismarkcouldbeawardedforanappropriatetreediagram.
Establishmentoftherelationship.

Considerationofonecasefollowing .
Thismarkcouldbeawardedforanappropriatetreediagram.
Establishmentoftherelationship.

Attempttosolvethesimultaneousequations.
Correctvaluesfor , and .
Attempttocombineprobabilitiestoobtainoverallprobabilityofwin.
Correctprobability.

Question13

(i)

for
for

B1

M1M1
A1

Usethesubstitution

intheintegral:

B1

Therefore

If

1thenchoose

If

1thenchoose

(ii)

.
ln .

ln asitispositive.

Applyingintegrationdonebefore:
2
2

intheintegral:
2

M1A1

and,applyingtheintegrationalreadycompleted,

.
Therefore

Var

Var

.
2

Var
For

A1

0,

1
0,sothevariancedecreasesas increases.

Var

M1

Var

A1

M1A1

B1

Usingintegrationbyparts:

B1

M1A1

0astheminimumoccursatanegativevalueof .

Usethesubstitution

A1

,sothestationarypointoccurswhen

M1

M1
A1

Question13
B1 Statementofrandomvariable.
M1 Anycorrectterminexpectation(allow multipliedbyanattemptattheprobabilityfor
notneedinganyextracosts).
M1 Correctintegralstated(allow missing).
A1 Fullycorrectstatement.

Maybealteredtoaccommodateothermethodsoncesolutionsseen.
B1 Substitutionperformedcorrectly.
M1 Integrationbypartsusedtocalculateintegral.
A1 Correctlyjustifiedsolution.
Notethatanswerisgiveninthequestion.
M1 Differentiationtofindminimumpoint.
A1 Correctidentificationofpoint.
B1 Bothcasesidentifiedwiththesolutionsstated.

M1 Attemptat
(atleasttwotermscorrect).
A1 Correctstatementof
.
B1 Substitutionperformedcorrectly.
M1 Applyingintegrationbyparts.
A1 Correctintegration.
A1 Correctexpressionfor
.
M1 UseofVar

A1 CorrectsimplifiedformforVar

M1 DifferentiationofVar
.
A1 Correctinterpretation.
Notethatanswerisgiveninthequestion.

1.(i)
1

1
1

B1
1
1

1
2

integratingbyparts

M1A1

A1*

(4)

(5)

1
2

1
1

M1

M1

B1

tan

Thus

M1
A1*

(ii)
.

usingthesubstitution

andthenthesubstitution

So
Usingthesubstitution

1 M1A1*

M1A1*

(iii)

M1A1

M1

M1

A1(5)

So

!
!

M1A1*(6)

2.(i)True.

B1

B1

1000

If

1000,then1000

(ii)False.

1and

E.G.Let

2for odd,and

Then forwhichfor

Soitisnotthecasethat
(iii)True.

,i.e. 1000

,so1000

B1
2and

,nor

1for even.

B1

M1

,butnorisitthecasethat

M1A1(4)

A1(4)

B1

meansthatthereexistsapositiveinteger,say

,forwhichfor

meansthatthereexistsapositiveinteger,say

,forwhichfor

E1

E1
max

Thenif
for

B1

A1(5)

B1

B1

4.

B1

M1A1

B1

sobytheprincipleofmathematicalinduction,

2 forsomevalue

Assume
Then

,andso

(iv)True.

2 for

4,andthus

2 A1(7)

3.(i)

If|
So

Symmetryaboutinitialline

Twobranches

Shapeandlabelling

sec |
,then
sec
sec

sec

or

G1
G1
G1(3)
or
sec

M1A1

Ifsec
0, sec
0as
and sec
arebothpositive,andthusinbothcases,
0whichisnotpermitted.
Ifsec

0, sec

sosec

0asrequired.

So
sec
thepole(origin),
dbeingb,

0and sec

B1

0giving

0as and
B1
0

(4)

,thuspointssatisfying(*)lieonacertainconchoidofNicomedeswithAbeing

B1

sec .

andLbeingtheline

B1

B1(3)

(ii)

Symmetryaboutinitialline
Twobranches

Loop,shapeandlabelling

G1
G1
G1


If

sec

,thenthecurvehastwobranches,

0,theendpointsoftheloopcorrespondingtosec

withsec

0and

withsec
.

B1(4)

1and

Inthecase

2so

2,sec

Areaofloop
2

sec
sec

4 sec
3

4 ln

4
2

tan

M1A1
4 ln|sec
3

tan |
4 ln 2

4
3

M1A1
M1A1(6)

sec

4.

(i)

For

andfor ,

iscontinuous.
.

B1

Sothesketchofthisgraphmustbeoneofthefollowing:

B1
Hence,itmustintersectthe axisatleastonce,andsothereisatleastonerealrootof
0

B1(3)
M1

(ii)
Thus

A1

0,

and,as

A1

0,

addingthesethreeequationswehave,
3

0 M1

(Alternatively,

So
Thus6

M1

A1*(6)

(iii)

cos

Let

cos 2

Then
As

sin

for

sin 2

and

1, 2, 3

cos 3

sin

M1
sin 3

bydeMoivre

M1

sin 2

sin 3

0
andso , ,and arereal,

M1

andthereforesoare , ,and

A1
0with , ,and real,bypart

Hence,as , ,and aretherootsof


(i),atleastoneof , ,and isreal.
cos

Soforatleastonevalueof ,
andas
If

M1
sin

0asrequired.

A1(6)

0(say
and

0)
andsothequadraticofwhich and arethe

rootshasrealcoefficients.Thus ,
If

Thuscos
Butsin
If

0then isreal. and aretherootsof

whichis

0,

cos

,andso

sin

andso

0,

isrealandthus,sin

.(

,as
.

0because

0)

M1

M1A1

0,andthus
B1(5)

B1

0,then and arerealroots,sosin


.

sin

0,so

,where ,

5.(i)Havingassumedthat2isrational(step1),2

0B1

,so provingstep3.

Thusfromthedefinitionof (step2),as and2


B1(2)
If ,then isanintegerand 2isaninteger.
1

So 2

isaninteger,

and 2

2andsoM1

B1
B1

2whichisanintegerandso 2

1,andthus0

provingstep5.B1(3)

A1

andthusthiscontradictsstep4that isthesmallestpositiveintegerin as 2
showntobeasmallerpositiveintegerandisin .

A1(3)

(ii)

If2

So 2

,thatis2

andhence2

andso2

,whichcanbewritten2

,where ,

0
2

M1

0M1

isrationalandthat2

provingthat2

hasbeen

isrational. A1

provingthat2

isrational,then2

If2

,where ,

isrational,then2

isrationalonlyif2

isrational.

A1(4)
Assumethat2

isrational.

Definetheset tobethesetofpositiveintegerswiththefollowingproperty: isin ifandonlyif


2

and 2

areintegers.

B1
,where ,

Theset containsatleastonepositiveintegerasif2
2

and

,so

Define tobethesmallestpositiveintegerin .Then 2


Consider

1 .

itselfaninteger.
and

1
2

0,then

M1A1

areintegers. B1

whichisthedifferenceoftwointegersandsois

2
2

and 2

2
2

whichisaninteger,
2

whichisaninteger.

Thus

1 isin .
2andso0

M1A1
1

1,andthus0

that isthesmallestpositiveintegerin as
integerandisin .

M1A1(8)

,andthusthiscontradicts

1 hasbeenshowntobeasmallerpositive

6.(i)

For ,

B1

,werequiretosolve

M1

2
2

0
8

2
2

4
2
2

Sofor,

,as

i.e.

so3

soif

,then 2

,
1

1or

Thus
Soas

0,

M1

M1A1

For ,

,from(i)werequire
.

inotherwords

1inwhichcase 1

whichitis,

1,

thediscriminantis

,so
0so

3,i.e.

M1A1(12)
whichitis,and

M1

So and neednotbereal.Acounterexamplewouldbe
forthen

M1A1

0,thevaluesof arereal.B1

and

Therearethreedistinctvaluesof unless

M1A1

Thusif
Thus

M1A1
3

mustbereal,and2

B1*(5)
2

M1A1

mustbereal,

(ii)

,i.e.2
B1(3)

B1
2

0inwhichcase

7.

(i)
Suppose
forsomeinteger .

M1

M1A1(3)

isapolynomialofdegree i.e.

B1

Then
1

whichisapolynomialofdegree .

Suppose

1,

istruefor

whichisdivisibleby 1

Wehaveshownthatifitistruefor
foranypositiveinteger
.B1(6)

for

sotheresult

1.Hencebyinduction,itis

i.e.
B1

,itistruefor

B1(6)

M1A1

isdivisibleby 1
1.

Then

,then

andwehaveshownthatifitistruefor
trueforanypositiveinteger.

1
(ii)
Suppose
someinteger ,with

M1A1

.M1A1
1.M1A1

soresultistruefor

1.Hencebyinduction,itistrue

,itistruefor

(iii)

M1

So

1
Butby(ii),
thusif
1,

isdivisibleby 1
1
0,andhence

1
andso

M1A1*(5)

M1A1
1

,and

cos

8.(i)

sin

sin

and

cos

Thus

sin

0,

cos

cos

sin

M1A1

sin

becomes

Thatis sin
as

cos

M1A1
cos

sin

sin
cos

sin

cos

cos

M1

Multiplyingoutandcollectingliketermsgives
cos
whichis

0.

So

andthus

sin

sin

M1

A1

A1

cos

M1A1*(7)

G1(4)
M1soln| |

(oralternatively

A1andhence

A1)

(ii)

becomes

sin

cos

cos

sin

cos

sin

cos

sin

sin

thatis
sin

cos

cos

cos

sin

sin

cos

Multiplyingoutandcollectingliketermsgives
cos

cos

sin

0.

whichis

sin

sin

cos

0 M1

A1

So
ln

A1

A1

M1

So
1

with

0
1
1

thatis

A1*

G1G1G1(9)

9.

Iftheinitialpositionof is,thenattime ,
1
2

1
2

,soconservingenergy,

M1A1A1

Thus,

M1

i.e.

Thegreatestvalue, ,attainedby ,occurswhen

A1*(5)

0.

M1

Thus
So

(negativerootdiscountedasallquantitiesarepositive)

Thus
2

and
2

M1A1(3)

As

differentiatingwithrespectto
2

1
2

2
M1A1

Thus

A1

Sowhen

,theaccelerationof is
2

M1A1(5)

Thatis

andthus
1

where istheperiod.

M1A1

So
1

1
Let

B1

then

andso

2
as

Thus
1

M1A1

andso
4

1
asrequired.

32

M1A1*(7)

10.

Thepositionvectoroftheupperparticleis
sin
cos

B1B1

sodifferentiatingwithrespecttotime,itsvelocityis
cos
sin

E1*(3)

Itsacceleration,bydifferentiatingwithrespecttotime,isthus
sin
cos

cos
sin

M1A1A1

sobyNewtonssecondlawresolvinghorizontallyandvertically
sin
cos

sin
cos

cos
sin

M1A1

Thatis
cos
sin

sin
cos

sin
cos

Theotherparticlesequationis
sin
cos

cos
sin

sin
cos

B1(6)

Addingthesetwoequationswefind
2
i.e.

0and

M1A1*

Thus
cos
sin
i.e.

cos

sin

sin and

sin
cos
sin

sin
cos
cos

cos

Multiplyingthesecondofthesebysin andthefirstbycos andsubtracting,

0andso

Thus

0.

andasinitially2

Thereforethetimetorotateby
As

andinitially

M1A1*(4)
,

isgivenby

,attime,

therodisinitially abovethetable.

M1A1
,so

,andso

A1
asthecentreof

M1A1

Hence,giventheconditionthattheparticleshitthetablesimultaneously,
0
/
1/2
/

Hence0

,or2

asrequired.M1A1*(7)

11.(i)Supposethattheforceexertedby ontherodhascomponents perpendiculartotherod


and paralleltotherod.Thentakingmomentsfortherodaboutthehinge,
0,M1
0yields

whichas
A1*(2)

0andhencetheforceexertedontherodby isparalleltotherod.

sin

Resolvingperpendiculartotherodfor ,
sin ,

Dividingby
cot

Thatis

sin

sin

cos

M1A1

cot

cos orinotherwords cot

cos asrequired.

M1A1*(4)

Theforceexertedbythehingeontherodisalongtherodtowards , B1
andifthatforceis ,thenresolvingverticallyfor , cos
sec .

so

M1A1

A1(4)

(ii)Supposethattheforceexertedby ontherodhascomponent perpendiculartotherod


towardstheaxis,thattheforceexertedby ontherodhascomponent perpendiculartothe
rodtowardstheaxis,

B1
thenresolvingperpendiculartotherodfor
M1A1
andsimilarlyfor

sin

sin

sin

sin

cos

cos

M1A1
0

Takingmomentsfortherodaboutthehinge,
Somultiplyingthefirstequationby
sin

sin
sin ,

Thatis cot

cos ,where

M1A1

andaddingwehave

sin

Dividingby

,thesecondby

cos

sin

cot

cos

M1A1

A1(10)

cos

12.(i)Theprobabilitydistributionfunctionof is

1
6

1
6

1
6

1
6

1
6

1
6

sotheprobabilitydistributionfunctionof

is

1
6

1
6

1
6

1
6

1
6

1
6

andthus

B1

Theprobabilitydistributionfunctionof is

2
1

36

3
2

36

4
3

5
4

36

36

6
5

36

36

36

10
3

36

36

11
2

12
1

36

M1
sotheprobabilitydistributionfunctionof

0
6

is

1
6

36

2
6

36

3
6

36

4
6

36

36

5
6

36

A1
whichisthesameasfor

andhenceitsprobabilitygeneratingfunctionisalso

Therefore,theprobabilitygeneratingfunctionof
andthustheprobabilitythat
(ii)

isalso

isdivisibleby6is1 6.

B1

B1(6)

Theprobabilitydistributionfunctionof is

1
6

2
6

1
6

1
6

1
6

. A1*

36

andthus

wouldbe

Thus

M1A1

exceptthatthepowersmustbemultipliedcongruenttomodulus5.

wouldbe

except

and

1
6

1
6

So

M1A1

1
5

B1

A1

1
6

M1A1*(8)

Thatis
1
6

1
6

is 1

Wenoticethatthecoefficientof insidethebracketin

So

asrequired.

5 1

1 1

5 1
6

4.

Soif isnotdivisibleby5,theprobabilitythat
whichinturnisthecoefficientof ,namely
If isdivisibleby5,theprobabilitythat

6
6

M1
where isan

However,thiscoefficientisthesumofaGPandso
integersuchthat0

1
5 1

43

1trivially.

Thiscanbeshownsimplybyinduction.Itistruefor
Consider

1
6

35

M1A1

isdivisibleby5willbethecoefficientof
1

isdivisibleby5willbe

asrequired.
1

as

B1*

Thatis

M1A1(6)

13.(i)

G1

if0

B1

G1
2

and
0if

So

0and

if1

1if

B1(5)
1
1

1
2
2

1

2
1

0

Soas

Thus

B1

M1A1

1
2
1
2

0
1

asrequired.

(ii)

1
2

1
2
1

M1A1*(4)

2 ln

1
2

2 ln

2 ln 2

M1A1(2)

G1
1

2
1
1
2

B1(2)

Thus

So
1 1
1
2
1 1
1

2

1
2

1
2

i.e.
1
3
2
1
2 1

1
2

1
2

1
2

Soas

1
2

1
1
2

1
2

1
2

M1A1(3)

M1A1(4)

B1

1
2

1
ln 1
2
1
2

1
1
2

1
2

asrequired.

and

1
2

because,bysymmetry,

M1A1

1 1
ln
2 2

1
ln
2
1 1
ln
2 2

1 1
ln
2 2

STEP Solutions 2015


Mathematics
STEP 9465/9470/9475
October 2015

The Admissions Testing Service is a department of Cambridge English


Language Assessment, which is part of Cambridge Assessment, a not-forprofit department of the University of Cambridge.
Cambridge English Language Assessment offers the worlds leading
qualifications for learners and teachers of the English language. Over 4 million
people from 130 countries take Cambridge English exams every year.

UCLES 2015
More information about STEP can be found at:
www.stepmathematics.org.uk

Contents
STEP Mathematics (9465, 9470, 9475)
Hints and Solutions
STEP Mathematics I
STEP Mathematics II
STEP Mathematics III

Page
4
10
15

SI-2015 Hints and Solutions


(See the marking scheme for full details of the expected solution approach)

Q1
This question is intended to be a relatively straightforward entre into the paper, and thus its
demands are fairly routine in nature. That does not mean that it is easy, merely that the appropriate courses
of action should be readily accessible to all candidates of a suitable standard. To begin with, the demand for
a sketch (of any function) should lead you to consider things such as
key points (such as where the curve meets either of the coordinate axes);
asymptotes (note the information given in italics at the end of part (i) regarding what happens as
x , which indicates that the negative x-axis is an asymptote in this case);
turning points of the curve, which are clearly flagged as being of significance when considering what
happens when y = k; i.e. when the curve meets a horizontal line;
long-term behaviour (you already have sorted for you the x side of things, so there is only a
quick decision to be made about what happens as x +).
For the key points, first set x = 0 and then y = 0; the asymptote is effectively given; the TPs come from
setting the first derivative to zero and solving for x again (noting, of course, that ex is always positive); and
the curve clearly grows exponentially as x increases positively. The rest of (i) then simply requires a bit of
thought as to how many times a horizontal line will cut, or touch, the curve depending upon the value of k.
In part (ii), it is clear that the x in part (i) has now been replaced with an x2, and this second curve
must therefore have reflection symmetry in the y-axis, as all negative values of x are being squared to give
the positive counterpart. Previously, when x was equal to zero, we now havex2 = 0, and so each previous
crossing-point on the positive xaxis leads to two, one on each side of the y-axis (and at the square-root of
its former value). However, the previous y-intercept is unchanged, but must now appear at a TP of the curve
(otherwise the symmetry of the gradient would be compromised). Also, the previous TP with positive xvalue (the negative one has gone) occurs at the square-roots of the previous value, but again with unchanged
y-coordinate.

Q2
It is clear that (i) is an introductory part that requires the use of the cos(A B) formula with suitablychosen values of A andB. Using the sin(A B) formula then leads to the second result, although there are
alternative trig. identities that could be used in both cases, such as a double-angle formula. Repeated use of
these, or the double-angle formulae (or de Moivres Theorem for those from a further maths background)
lead to the (relatively) well-known triple-angle formula cos3 4cos3 3cos, which gives x = cos as a
root of the equation 4 x 3 3 x cos 3 0 . Although there are several possible methods here, a simple

division/factorisation leads to ( x c) 4 x 2 cx c 2 3 = 0, and the quadratic formula leads to the


remaining two roots which, for their simplest form, requires the use of the most elementary of trig.
identities, s 2 1 c 2 .
A bit of insight is needed in part (iii), where one should first realise that the constant term is intended
be a cosine value (of 3 some angle), and the most obvious candidate is
From here, it is now clear that x is

1
2

1
2

2 = cos 45o, so that is 15o.

y , and that the three roots are those from part (ii) with the exact

numerical forms of the sine and cosine of 15o from part (i) waiting to be deployed in order to find the surd
forms requested.

Q3
In this question, it is important to draw suitable diagrams in order to visualise what is going on, and
these are not difficult to manage, with the guard either at a corner of the yard (C) or at its middle (M).
However, this second case has two possible sub-cases to consider, depending upon whether the far corners
of the yard are visible to him/her (which, in fact, turns out to be the b = 3a case which separates the two
cases that the question invites you to consider), and it is the extra length of the opposite wall that is visible
b2
4b 2
(from C),
(from M, with the far corners not
that makes for different working. These lengths are
ba
a
2b( 2b 3a )
visible) and
(from M, with the far corners visible). Once obtained, these should be compared
ba
in order to find that the guard should stand at C for b < 3a and at M for b > 3a (and at either when b = 3a).

Q4
A quick read-through of the question should make it clear that it is the lower end of the rod that is
being referred to (as do the subtractions within the given coordinates x and y). The fact that the rod is
tangent to the given parabola means that its direction is tan 12 x , which gives the coordinates of the rods
midpoint as (tan, tan2); a simple right-angled triangle and some accompanying basic trig. then leads to the
given answer. The second part of the question is equally straightforward once one realises that when xA = 0,
2 tan
2 tan b cos b
yA= tan 2 . There are several ways to attack the area between two
cos
curves e.g. as

y 2 dx, or by translating the bit below the x-axis up by tan2 and calculating the

difference between area under the new curve and a triangle; the key is to eliminate the b and then the
given answer follows.

Q5

Once one realises that the x within the integral is not the variable, then both integrations are

2x
1
, while in (ii) g(x) = 1 x 1 x remember to use
x
x
the modulus function when taking square-roots (although one could, alternatively, work out a piece-wise
definition for g; that is, in bits). The sketch of f should prompt the solver to differentiate in order to identify
1

the turning point at


, e ln 2 . Noting that y + as x 0 and that y + as x + gives all else that
ln 2

is needful to draw the graph in (i). In (ii), the piece-wise definition of g is certainly more useful now since its
graph is made up of two reciprocal curve bits joined by a horizontal straight-line in the middle.
relatively straightforward. In (i), we get f(x) =

Q6
The best way to start any geometrically-inclined question is to
have a good diagram that doesnt make the shape of the quadrilateral
look too specialised in any way (square, rectangle, parallelogram, ).
Next, label the midpoints sensibly (see diagram) and then write down
their position vectors in terms of a and b.
It is relatively easy to prove that the opposite sides of
this quadrilateral are equal and opposite, but you must then
also show that adjacent sides are equal in length and that they
are perpendicular. This last outcome is going to follow from the
S
use of the scalar product.
For the final part, you should label one of the angles at the

P
Q

centre (say) and note that the fourth angle at O is thus 180o .
Having already calculated the squares of the lengths of the squares sides in the form
2
o
1
2

4 a ( a ) 2 aa cos 90

the required result follows from noting that this is maximal when cos90

1 ; i.e. when 90 o .

Q7
The crucial observation here is that a (continuous) function takes its maximum value on a finite
interval either at a maximum turning-point or at an endpoint. Differentiating (a negative cubic so we
know what its shape is) gives a MIN. TP at (0, 0) and a MAX. TP at ( 13 a, 19 a3), and evaluating at the
endpoints gives f( 13 ) = 19 (3a + 2) and f(1) = 3a 6.
Now, a comparison of these possible values for f then yields that 19 (3a + 2) 19 a3 a 0, a 2; and
that 19 a3 3a 6 holds for all a 0; and also that 19 (3a + 2) 3a 6 a

7
3

(which, actually, affects

19 (3a 2) 0 a 2

nothing, but the working should be done anyhow). Thus M(a) = 19 a 3


2a3 .
3a 6
a3

Q8
The standard bookwork approach to this opening part is to write the sum (S) both forwards and
backwards, add the terms in pairs (n pairs, each of value n + 1) and then to half this to get S = 12 n(n + 1). As
with any such invitation to establish a result, one should not simply seek to quote a result and thus merely
write down the given answer. When looking at part (ii)s question, the binomial theorem should really be
screaming at you from the page, and all that is needed is to observe that the binomial expansion of (N m)k
consists of k + 1 terms, the first k of which contain a factor of (at least one) N. The final term, since k is odd
must be mk which then conveniently cancels with the + mk term to leave something that is clearly divisible
by N.
In the next part of the question, you are invited to explore the cases n odd and n even separately
(indeed the results that follow are slightly different). To begin with,
S = 1k + 2k + + nk (an odd no. of terms) = 0k + 1k + 2k + + nk (an even no. of terms)
So these terms can now be paired up:
n with 0, n 1 with 1, , ( 12 n 12 ) with ( 12 n 12 ) ,
so that all pairs are of the form (n m)k+ mk , which was just established as being divisible by n. Next, in the
case when
S = 1k + 2k + + nk (an even no. of terms) = 0k + 1k + 2k + + nk (an odd no. of terms),
the pairs are now
n with 0, n 1 with 1, , ( 12 n 1) with ( 12 n 1) ,
but with an odd term, ( 12 n) k , left over. This gives us (from the same previous result as before) a sum
consisting of terms divisible by n and one that is divisible by

1
2

n, giving the second result.

Then, for n even, so that (n + 1) is odd, S + (n + 1)k is divisible by n + 1 (by the previous result) S
is divisible by n + 1; and for n odd, so that (n + 1) is even, S + (n + 1)k is divisible by 12 (n + 1). Thus, since
hcf(n, n + 1) = 1 hcf( 12 n, n + 1) = 1 for n even, and hcf(n,
divisible by

1
2

n(n + 1) for all positive integers n.

1
2

(n + 1)) = 1 for n odd, it follows that S is

The standard time taken to land (at the level of the projection) of a projectile is t

Q9

bullet fired at time t, 0 t

2u sin
. Thus, a
g

2u

, lands at time TL t sin t . Differentiating this w.r.t. t and


g
6
3

2u 2 sin cos

setting it equal to zero, gives k cos t . The horizontal range is then given by R
and
g
3

this gives the required answer. Moreover, substituting the endpoints of the given time interval 0 t

dTL
1
3

0 throughout the
into k cos t gives k
. However, if k < 12 , then one sees that
dt
2
2
3

guns firing, so that TL is a (strictly) decreasing function. Hence its maximum value occurs at t = 0, i.e.

, whence R

2u 2 1
3 u2 3

.
g
2 2
2g

Q10 The difficulty in this question lies in ignoring unnecessary information (not given). Firstly, then, note
that the speed of the rain relative to the bus is vcos u (or u vcos if negative), and when u = 0, the area
of the bus getting wet, A, is such thatAhvcos + avsin . Now the given result follows from observing that
when vcos u> 0, the rain hitting the top of the bus is the same, whilethe rain hits the back of the bus as
before, but with speed vcos u instead of vcos ; and whenvcos u< 0, the rain hitting the top of the bus
is the same, while the rain hits the front of the bus as before, but with u vcos instead of vcos.
1
av sin h | v cos u |

. For vcos u > 0


Next, as the journey time , we need to minimise J
u
u
u
av sin hv cos

h , and this decreases as u increases, and this is done


and w vcos , we minimise J
u
u
av sin hv cos

h,
by choosing u as large as possible; i.e. u = w. For u vcos > 0, we minimise J
u
u
and this decreases as u increases if a sin > h cos , so we again choose u as large as possible; i.e. u = w.
Next, if a sin < h cos , then J increases with u when u exceeds vcos , so we choose u = vcos in this
case. Finally, if a sin = hcos then J is independent of u, so we may as well take u = w.
av sin hv cos

h , which
For the return journey, simply replace by 180o to give J
u
u
always decreases as u increases, so take u = w again.

Q11 As with all statics problems, the key to getting


a good start, and to making life as easy as possible
for the working that follows, is to have a good, clear
diagram with all relevant forces, in appropriate
directions, marked on it (see alongside).
To begin with, take moments about the
respective cylinders axes yields F = F1 = F2, as
required. Next, write down the four equations
that arise from resolving for each cylinder in the
directions parallel and perpendicular to the plane.

O1
O2

These are:F1 R W1 sin ; R1 F W1 cos ; F2 R W2 sin and R2 F W2 cos .


(Note that one could replace some of these with equivalent equations gained from resolving for the whole
system.) Replacing F1 and F2 by F, equating for sin , re-arranging for F in terms of R and using the
Friction Law, F R , appropriately leads to the first given answer in (ii). A bit more determination is
needed to gain the second given answer, however. Firstly, tan can be gained by division in at least two
ways, and both F and R must be eliminated from any equations being used. Thereafter, it is simply a matter
of forcing the working through correctly and, hopefully, concisely.

Q12 Here, you are given the relevant Poisson result at the outset, and this is intended to guide your
thinking later on in the question. To begin with, though, part (i) is actually a Binomial situation requiring
just a single general term. In part (ii), you were asked to prove algebraically a result that you might usually
be required to quote and use. This requires a good understanding of the use of the sigma-notation and a clear
grasp as to which of the various terms are constant relative to the summation, and then combining the
remaining terms together appropriately to give the requested Poisson answer. Most important of all, of
course, it is essential to have the first line of working correct; this is
r

e 8 8 n
n!
1 3

P(S = r) =

r ! (n r ) ! 4 4
nr n!

nr

e 8 2 r 6 n r
is obtained. At this stage

r ! n r (n r ) !
another simple trick is required effectively a re-labelling of the starting-point, using m = n r to re-write

and one follows this through to the point where the result

e 8 2 r 6 m
. The required result follows immediately since the infinite sum is just e6.

r ! m0 m!
Having established this, the final part of the question is relatively straightforward, requiring only the
use of the conditional probability formula applied to P(M = 8 |M + T = 12).
this as

Q13 The first three parts of this question are very easy indeed, if looked at in the right way. In part (i) it is
not necessary at all that you recognise the Geometric Distribution (indeed, some of you may not have
encountered it at all), but the result asked for is simply (n 1) failures followed by 1 success, and one can
write down immediately, and without explanation, the answer P(A) =

56 n 1 16 . In (ii), you have a situation

in which one can apply the principle of symmetry: either a 5 arises before a 6, or vice versa, so the required
probability is just P(B) = 12 . Part (iii) can be approached similarly, in that the first 4s, 5s, 6s can arise in the
orders 456, 465, 546, 564, 645, 654 P(BC) =

1
3

(i.e. also by symmetry, but with three pairings to

consider).
Parts (iv) and (v), however, each turn out to require the use of the result given at the end of the
question, as the outcomes (theoretically) stretch off to infinity. For (iv), it is best to consider only on which
throw the first 6 occurs (since we stop at that point). It cannot occur on the first throw, so we have the sum
of the situations:
a 5 occurs on the first throw, followed by a 6 on the second;
one 5 and a 1-4 occur, in either order, followed by the 6 on the third;
one 5 and two 1-4s occur, in any of three possible orders, followed by a 6 on the fourth;
etc.

Thus P(D) =

16 16 16 64 16 16 64 2 16 ... , and this factorises as 361 1 2 23 3 23 2 ..., and the


2
1

3
1

big bracket is just the given result with x =

2
3

and n = 2.

Before getting too deeply into part (v), a couple of simple results should be noted. Firstly, we use the
fact that P(E) = P(D) = 14 , the answer to (iv); and then that we will need to use the basic probability result
P(DE) = P(D) + P(E) P(DE) =

1
2

P(DE). Turning this around, since it is far easier to calculate the

probability, P(DE), that both one 4 and one 5 occur before the first 6. Again, looking at this from the
viewpoint of finishing after the first 6 is thrown, we see that
3
4 2
1
1 3 12 6 12 2 ...
P(DE) = 62 16 16 63 62 16 16 63 62 16 16 ... = 108
1
2

and the big bracket is the given result with x =

1
2

and n = 3 , leading to the answer P(DE) =

23
54

STEP22015HintsandSolutions
Question1
Forthefirstresult,showthatthegradientofthefunctionispositiveforallpositivevaluesof (by
differentiating)andalsothat 0
0.Oncethisresulthasbeenestablishedsumasetoftheterms,
using

,notethatln 1

canbewrittenasln

ln

andthentherequiredresult

follows.
Forthesecondpart,firstshowthat
substitution

,notingthatln 1

withthesumstartingwith

ln 1

1andthenusethe

isnegativefor0

canbewrittenasln

2 ln

ln

1 .Deal

2andthenaddtheinitial1afterwards.

Question2
Aswithallgeometricquestionsagooddiagramoftheinformationgivenmakesthesolutiontothis
questionmucheasiertoreach.Thefirstresultinthisquestionfollowsfromanapplicationofthesine
rulewithapplicationsoftherelevantformulaeforsin
andthedoubleangleformulae.From
.
adiagramofthetriangleitshouldthenbeaneasyapplicationoftrigonometrytoshowthat
Thereareanumberofdifferentmethodsforestablishingthat
methodistoshowthatsin
thesizesofangles
relationship.

trisectstheangle

one

,followingwhichitisrelativelystraightforwardtoworkout

and

intermsof andshowthattheymustsatisfythecorrect

Question3
canbeinterpretedasthetrianglesthatcanbemadeusingthe
Forthefirstpartnotethat
rodoflength8andtwoother,shorterrods.Thesecanthenbecountedbynotingthatthereare6
possibilitiesifthelength7rodisused,4possibilitiesifthelength6(butnotthelength7)rodisused
and2possibilitiesifthelength5(butnot6or7)rodisused.Itisclearthatatleastonerodlonger
thanlength4mustbeused.Toevaluate
and
notethatitisequalto
thenevaluate
inasimilarmannerto
.Similarreasoningeasilygivesformulaefor
and
.
deducedinthepreviouspartcanbeusedtoshowthe
Fortheinduction,therulefor
inductivestep,whiletheeasiestwaytoshowthebasecaseistolistthepossibilities.Theeasiestway
toestablishtheresultforanoddnumberofrodsistousetheformulafor
andthe
formulafor
thatwasjustproven.

Question4
Forthefirstpart,notethatthegraphofarctan satisfiestherequirementofbeingcontinuous,but
doesnotsatisfy 0
.Sincetan
tan ,atranslationofthegraphof
arctan
verticallybyadistanceof givestherequiredgraph.
Itshouldbeclearthatthegraphof

hasnoverticalasymptotes,approachesthe axisas

andpassesthroughtheorigin.Identifyingthestationarypointsshouldbethenexttask
afterwhichagraphshouldbeeasytosketch.Thegraphof
shouldthenbeeasytosketch
byconsideringthefactthat
isanincreasingfunctionand
isobtainedbycomposingthe
twofunctionsalreadysketched.
Tosketchthegraphof

firstnotethattheremustbetwoverticalasymptotes.Once

stationarypointshavebeencheckedforitshouldbestraightforwardtocompletethesketch.Inthis
case,theasymptotesneedtobeconsideredtodeducetheshapeofthegraphfor
asthe
compositionwith
willleadtodiscontinuities.Notingagainthattan
tan the
discontinuitiescanberesolvedbytranslatingsectionsofthatgraphverticallybyadistanceof .

Question5
Theinitialproofbyinductionisastraightforwardapplicationofthetan
formula.Thefinal
partofsection(i)requiresrecognitionthattherearemanypossiblevaluesof togiveaparticular
valueoftan ,butonlyoneofthemisthevaluethatwouldbeobtainedbyapplyingthearctan
function.Theresultcanthereforebeshownbyestablishingthatthedifferencebetweenconsecutive
termsofthesequenceisnevermorethan .
Forthesecondpartofthequestionadiagramofthetriangleandapplicationofthetan 2 formula
showsthatthevalueof mustbeoftheformusedinthefirstpartofthequestion.Allthat
remainsisthentoshowthatthelimitofthesummustgivetherequiredvalue.

Question6
Thefirstpartofthequestionrequiresuseofthecos
formula.Followingthistheintegral
tan
.Inthesecondpart,applythe
shouldbeeasytoevaluategiventhat sec
substitutionandnotethatthelimitsoftheintegralarereversed,whichisequivalenttomultiplying
by1.Followingthisasimplerearrangement(notingthatthevariablethattheintegrationistaken
overcanbechangedfrom to )shouldestablishtherequiredresult.Theintegralattheendofthis
partcanthenbeevaluatedsimplybyapplyingthisresultalongwiththeintegralevaluatedinpart(i).
Inthefinalpartofthequestionitistemptingtomakerepeatedapplicationsoftheresultprovenin
part(ii).However,thisisnotvalidasitwouldrequiretheuseofafunctionsatisfying sin
,
whichisnotpossibleontheintervaloverwhichtheintegralisdefined.Instead,applicationofa
sin
willsimplifytoallowthisintegraltobeevaluated
similarsubstitutiontopart(ii)to
basedontheintegrationof

.Anapplicationoftheresultfrompart(ii)willalsoberequired.

Question7
Forpart(i)notethatthelinesjoiningthecentresofthetwocirclesandoneofthepointswherethe
bisectionoccursformarightangledtriangle,sotheradiusofthenewcirclecanbecalculated.To
showthatnosuchcirclecanexistwhen
notethatthediametricallyoppositepointson must
beadistanceof2 apart,andnotwopointsonacircleofradius canbethatfarapart.Forthecase
notethatthenewcirclewouldbethesameas (andsowouldhavemorethantwo
intersectionpoints).
Forpart(ii)asimilarmethodcanbeusedtodeducethedistancesbetweenthecentreofthenew
circleandeachof and .Fromthesedistancesequationscanbeformedrelatingthe and
coordinatesofthecentreofthenewcircle.Itisthenaneasytasktoeliminatethe coordinateof
thecentreofthecirclefromtheequationstogetthegivenvalueofthe coordinate.
Theexpressionfor caneasilybefoundbysubstitutingbackintotheequationsobtainedfromthe
0toobtainthe
distancebetweenthecentresoftwoofthecircles.Oncethisisdone,notethat
finalinequality.

Question8
Thefirstpartofthequestionfollowsfromconsiderationofsimilartrianglesinthediagramiftheline
through andthecentresofthecirclesisadded.Forthesecondpart,expressionscanbewritten
downforthepositionvectorsof and bynotingthatthesamemethodasinpart(i)willstillapply.
Thevectors and canthenbecomparedtoshowthatoneisamultipleoftheother.
Forthefinalpartofthequestion,notethat willliehalfwaybetween and if

Question9
Adiagramtorepresentthissituationwillshowtheanglesthatwillberequiredtocalculatethe
momentsofeachoftheparticlesabout intermsof .Followingthis,simpletrigonometric
manipulationshouldleadtoarelationshipbetweensin andcos .Fromthis,eitherarightangled
triangleoroneofthebasictrigonometricidentitiescanbeusedtoreachtherequiredresult.
Forthesecondpartofthequestiontheamountofpotentialenergythatneedstobegainedbythe
systemshouldbeeasytocalculateandthismustbeequaltotheinitialkineticenergyofthesystem.

Question10
Thecomponentofthevelocityoftheparticleinthedirectionofthestringatanymomentmustbe
equalto ,whichleadsto cosec asthespeedoftheparticlealongthefloor.Alternatively,
introduceavariabletorepresentthelengthofstringstillintheroomortheheightoftheroomand
thendifferentiate ,thedistanceoftheparticlefromthepointdirectlybeneaththehole,with
respecttotime.Thelengthofthestring(totheholeintheceiling)isdecreasingatarateof
,
whichthenallowstheintroducedvariabletobeeliminatedtoreachanexpressionforthespeedof
theparticle.
Differentiationofthespeedoftheparticleallowstheaccelerationtobecalculated.Finally,notethat
theparticlewillremainontheflooraslongastheverticalcomponentofthetensionislessthanthe
weightoftheparticleandthenthepointatwhichtheparticleleavesthefloorcanbeidentified.

Question11
Forthefirstpart,thecoordinatesof arefoundbyapplyingsimpletrigonometricratiosand
differentiationwithrespecttotimegivesthevelocityof .Inthesecondpart,thefirstequation
resultsfromconsiderationofconservationofmomentumandthesecondresultsfromconservation
ofenergy(withasubstitutionbasedonthefirstequationmadetoeliminateonevariable).
Sincenoenergyislostinanycollisionstherelationshipsfrompart(ii)mustcontinuetoholdandthis
showsthat cannotbe0whichmeansthatthedirectioninwhich changesremainsthesame
unlessthereisacollision.Sincethefirstcollisionoccurswhen
0,thesecondonemustbewhen
.
Forthefinalpart,notethattheequationsinpart(ii)muststillhold,andif
0,thekineticenergy
of mustbe0.Sincethekineticenergiesof and mustbeequal(bysymmetry)itmustbethe
casethatthekineticenergyof is

andcanalsobecalculatedfromtheexpressionforthe

velocityof showninpart(i).Since
0,thiscanthenbeusedtofindthevaluesof .Finally,
notethatgiventhesevaluesof , willonlybe0ontheoccasionswhen ispositive.

Question12
Forthefirstpart,notethat canonlywinthegameifthefirsttwotossesresultinheads,sinceonce
therehasbeenatail, willwinassoonastwoconsecutiveheadshavebeentossedand cannot
winuntiltherehavebeentwoconsecutiveheadsandonefurthertoss.Inthesecondpart,notethat
thislogicstillappliestothegamefor andsimilarreasoningcanbeappliedtothegamefor .For
theothertwoplayersswitchingheadsandtailsinanysequencethatresultsinawinfor willgivea
sequencethatresultsinawinfor ,andviceversa,sotheprobabilitiesmustbeequal.Sinceonly
sequenceswhichalternatebetweenheadsandtailsforever(andtheprobabilitiesofsuchsequences
tendtozeroasthelengthsofthesequencesincrease)theprobabilitiesmustbothalsobe.
Forthefinalpart,notethat mustwinifthefirsttwotossesareTT.Sinceonlytheprevioustwo
tossesareimportantindeterminingwhatcouldhappenonthenexttoss,eachcasecanbeanalysed
byatreediagramwhichshowstheoutcomesafteronefurthertoss.
Forexample,followingHT:

HgivesthepositionifthelasttwotosseswereTH,andsoaprobabilityofwinningof ,
TgivesthepositionifthelasttwotosseswereTTandsoaprobabilityofwinningof1.

Thetotalprobabilityistherefore

,butthismustalsobeequalto .

Thisyieldsthreeequationsinthethreeunknownswhichallowsalloftheindividualprobabilitiesto
becalculated.Oncethisisdonetheoverallprobabilitycanbecalculated.

Question13
Tocalculatetheexpectedvalueofthetotalcost,notethatthereisaconstantcomponentof and
thentheexpectedvalueofthe
giventhat
mustbeadded,whichcanbecalculated
byintegrationof
withrespectto ,between and.Differentiatingtheexpression
for
withrespectto allowsthepositionofthestationarypointtobefound.Ifthisisata
negativevaluethen shouldbechosentobe0andotherwisethevalueof forthestationarypoint
shouldbeused.
Aslightlymorecomplicatedintegrationisneededtoestablishtheformulafor
andthen
differentiationofthisgivesavaluethatisclearlynegativeforpositivevaluesof ,whichshowsthat
thevarianceisdecreasingas increases.

STEP32015
1.

Hintsandsolutions

ThefirstresultcanbeobtainedbysimplifyingtheLHSandthenwritingitas
andintegratingthisbyparts.Toobtaintheevaluationof
,thefirstresult

canberearrangedtomake
thesubject,andtheniteratingtheresulttoexpressitintermsof
whichisastandardintegral.Theexpressioncanbetidiedbymultiplyingnumeratorand
denominatorby 2 2
2 2 .Thefirstresultfor(ii)isobtainedbymeansofthe
substitution
,thesecondbyaddingthetwoversionsof ,andthethirdbythesubstitution
,beingcarefulwithlimitsofintegrationandemployingsymmetry.Part(iii)issolvedby
andthenemployingfirstpart(ii)thenpart(i)toobtain

expressingtheintegrandas
!

,whichis

2.
Part(ii)istheonlyfalsestatement,andasimplecounterexampleis
1and
2for
odd,and
2and
1for even.Part(i)
1000isasuitablevalue,then1000

andas ispositive,theinequalitycanbemultipliedbyitgivingtherequiredresult.Part(iii)
requirestheuseofthedefinitiontwicewithvalues and say,andthenusing
max
.Forpart(iv),wecanchoose
4,andaninductiveargumentsuchas
,
1

3.

Thepart(i)inequalityforsec canbeobtainedbymaking thesubjectoftheformulaas


sec
andinvoking
rememberingthat
0isnotpermitted.

works.

ThenthepointslieonaconchoidofNicomedeswithAbeingthepole(origin),dbeingb,andLbeing
theline
sec (" "
).Asketchis

Inpart(ii),theextrafeatureistheloopasspecifiedwithendpointsatthepolecorrespondingto
sec

.Asketchis


sec

Sointhegivencase,theareaisgivenby2

whichis

4 ln 2

3 .

4.
Part(i)isimplyshownbyconsideringtheimageofthefunction
as andthenobservingthatthefunctioniscontinuousandexhibitsasignchange.Part(ii)
giving
canbeapproachedbywriting
,

,whichcanbeobtainedbyconsidering

andtherequiredresultfor6 which
0.

canbeneatlyobtainedbyconsidering

cos
for
1, 2, 3,employingdeMoivrestheorem,thethree
sin
Writing
sumsimplytherealityof , ,and ,andhence , ,and whichbyvirtueoftheresultof
part(i)yieldstherealityof , ,or andhencetherequiredresult.Thefinalresultcanbe
consideredastwocases,thetrivialoneofallthreerootsbeingreal,andtheonewheretheother
twoarecomplex.Thelattercanbeshowntogivetherequiredresultbyconsideringtherealand
imaginarypartsoftherootsofarealquadratic.
5.
(i)Step3isstraightforwardonthebasisofsteps1and2,notingthatnolowestterms
restrictionneedbemadeinpart1.Step5requiresthatthegivenexpressionisapositiveintegeras
wellaswellasbeingintegerwhenmultipliedbyroottwo.Step6requiresjustificationthat
2 1 1.

(ii)Therationalityof2 onthebasisof2 beingrationalissimplyobtainedbysquaring


thelatter,andtheoppositeimplicationcanbemadebysquaringtheformerordividing2bythe
former.Toconstructthesimilarargument,lettheset bethesetofpositiveintegerswiththe
followingproperty: isin ifandonlyif 2
smallestpositiveintegerinthatset,consider
6.

and 2
2

areintegers,andtaking tobethe

1 toproducetheargument.

Treatingtheequationsfor and assimultaneousequationsfor and ,onefindsthat

and

whichdemonstratesthatif

and

2 ,i.e.

0.

and arereal.If and arereal,then and are(trivially)and2


In(ii),thefirsttwoequationsyield3
1

1,makingitpossibletowritethethirdequationas

1 whichhasanobviousfactorof

1 leadingto

1or

fromthequadraticequation.Ifoneofthesolutionsofthequadraticequationgivesthesameroot
1,thentherearenotthreepossiblevalues,i.e.if
2.Fromthefirstpartofthequestion,
for and tobereal,wewouldwant tobereal,

tobereal,and

,in

otherwords

.Soacounterexamplecouldbe

1giving2

0whichhasa

negativediscriminant.
with
Theopeningresultissimplyachievedbyfollowingthegivenexplanationfor
.Parts(i)and(ii)canbothbeshownusualtheprincipleofmathematicalinductionwith
initialstatements
7.

Suppose
isapolynomialofdegree i.e.

for
someinteger .andSuppose
1
1
isdivisibleby 1
i.e.
1
forsomeinteger ,with
1.Part(iii)isobtainedbyexpressing
1
1
insigmanotation(bythebinomialtheorem),thencarryingout
usingthe
ideainthestem,andfinallyinvokingtheresultofpart(ii)andthensubstituting
1.
8.

Transformingthedifferentialequationinpart(i)ismadebysubstitutingfor and as

given,for using

cos

sin andasimilarresultfor ,andthensimplifyingthe

algebrabymultiplyingoutandcollectingliketermsbearinginmindthatafactor canbecancelled
as
0.Thetransformedequationcanbesolvedbyseparatingvariablesorusinganintegrating
factor,togive
,thesketchofwhichis

0whichissolvedby

Thesametechniquesforpart(ii)yieldsadifferentialequation

separatingthevariablesandthenemployingpartialfractionsgivingavarietyofpossiblesolution
sketches

1butitispossibletoconsider

9.

Whilstthefirstpartcanbeobtainedotherwise,thesimplestapproachisbyconserving

energy,when
foundbysetting

0leadingto

1inwhichcase

0)

leadstotherequiredanswersimply. is
.Theaccelerationcanbefoundbyapplying

Newtons2ndLaworbydifferentiatingtheequationfoundinthefirstpart,andsubstitutingleadsto

theresult

fortheaccelerationwhen

.Treatingtheequationfoundinthefirst

partasadifferentialequationfor intermsof,theexpressionfortheperiodis

.Makingthesubstitution

,leadsto

2
approximate

,whichmakingabinomialexpansionandusingthegivenconditionto
2

resultsinthefinalgivenexpression.

sin
sodifferentiatingwithrespectto
cos
timeyieldsthecorrectvelocityandaccelerationwhichgivesthesecondresultwhenusedin
Newtonssecondlawresolvinghorizontallyandvertically.Thecorrespondingequationsare
cos
sin
sin
0
fortheotherparticlebymerelyswappingthe
cos
1
sin
cos
directionofthetensionandthedisplacementfromthemidpoint.Thedeductionsareobtainedby
addingthetwoequationsofmotion,andinthecaseof,subtractingthetwoequationsandthen
cos
eliminating betweentheequationsforeachcomponent.Using
anda
sin
similarequationforthelowerparticle,initialvaluesof and canbefoundandthenthetimefor
10.

Thepositionvectoroftheupperparticleis

therodtorotateby

canbeobtainedandsubstitutedinthedisplacementequationunder

uniformaccelerationtoobtainthefinalresult.
11.
(i)Thefirstresultisobtained,asthequestionprompts,byconsideringacomponentofforce
ontherodduetoP,andtakingmomentsaboutthehingetofindthatthatcomponentiszerowith
theconsequencethatanyforceexertedontherodbyPmustbeparalleltotherod.Bearinginmind
thehorizontalaccelerationofPtowardsthecentreofthecircleitdescribes,resolvingperpendicular
totherodandwritingtheequationofmotionforPleadsdirectlytothegivenequationwiththe
statedsubstitutionbeingmade.Theforceexertedbythehingeontherodisalongtherodtowards
PandresolvingverticallyforforcesonPandrearranginggives
sec .

(ii)Takingmomentsforthewholesystemaboutthehingegives


sin

sin

sin

cos

sin
.

whichcanberearrangedintotherequiredformwith
12.

cos

(i)Therequiredprobabilitygeneratingfunctionis

anditissimpletowritedowntheprobabilitydistributionfunctionof andhenceof andarrive


atthesamepgf.Asaconsequence,itcanbearguedthatthepgffor isalso
andsothe
1
requiredprobabilityis 6.

(ii)

wouldbe

thatthepowersmustbemultipliedcongruenttomodulus5,anditcanbeshownthat
5 soobtainingtherequiredresultfor

.Obtaining

except
and

where isanintegersuchthat0
5
4,andtheprobabilitythat isdivisibleby5will
bethecoefficientof whichinturnisthecoefficientof asrequired.If isdivisibleby5,the
probabilitythat
13.

isdivisibleby5willbe

as

(i)

leadto
0if

1,

if0

0and

1if

2,

2.
bymeansofthelogic

Fromthis,thecumulativedistributionfunctionof
1

andtherequiredprobabilitydensity
2 ln 2

functioncanbefoundbydifferentiation.Fromthat,byintegration,
(ii)

if1

leadto
1

2
1
1
2
1

That

probabilitydensityfunctionfound.

1 anddifferentiationleadsto

1
0

canbewrittendownbecausebysymmetry,
and

theprobabilitydensityfunction

1.Thisissimplyverifiedbyintegrationusingthe

[Anearlierpotentialversionofthequestionhad , ,and independentlyuniformlydistributed


on 0,1 ,consideredthedistributionofln ,
wentontofindthepdfof ,where
uniformlydistributedon 0,1 .]

ln

andfinishedbyshowingthat

isalso

You might also like